Anda di halaman 1dari 91

Situation 1 - Mr.

Ibarra is assigned to the triage area and while on


duty, he assesses the condition of Mrs. Simon who came in with asthma.
She has difficulty breathing and her respiratory rate is 40 per minute.
Mr. Ibarra is asked to inject the client epinephrine 0.3mg
subcutaneously

1. The indication for epinephrine injection for Mrs Simon is to:

a. Reduce anaphylaxis
b. Relieve hypersensitivity to allergen
c. Relieve respirator distress due to bronchial spasm
d. Restore client’s cardiac rhythm

2. When preparing the epinephrine injection from an ampule, the


nurse initially:

a. Taps the ampule at the top to allow fluid to flow to the base of the
ampule
b. Checks expiration date of the medication ampule
c. Removes needle cap of syringe and pulls plunger to expel air
d. Breaks the neck of the ampule with a gauze wrapped around it

3. Mrs. Simon is obese. When administering a subcutaneous


injection to an obese patient, it is best for the nurse to:

a Inject needle at a 15 degree angle' over the stretched skin of the


client
b. Pinch skin at the Injection site and use airlock technique
c. Pull skin of patient down to administer the drug in a Z track
d. Spread skin or pinch at the injection site and inject needle at a 45-
90 degree angle

4. When preparing for a subcutaneous injection, the proper size of


syringe and needle would be:

a. Syringe 3-5ml and needle gauge 21 to 23


b. Tuberculin syringe 1 mi with needle gauge 26 or 27
c. Syringe 2ml and needle gauge 22
d. Syringe 1-3ml and needle gauge 25 to 27

5. The rationale for giving medications through the subcutaneous


route is;

a. There are many alternative sites for subcutaneous injection


b. Absorption time of the medicine is slower
c. There are less pain receptors in this area
d. The medication can be injected while the client is in any position
Situation 2 - The use of massage and meditation to help decrease stress
and pain have been strongly recommended based on documented
testimonials.

6. Martha wants to do a study on, this topic. "Effects of massage


and meditation on stress and pain." The type of research that best
suits this topic is:

a. applied research
b. qualitative research
c. basic research
d. quantitative research

7. The type of research design that does not manipulate


independent variable is:

a. experimental design
b. quasi-experimental design
c. non-experimental design
d. quantitative design

8. This research topic has the potential to contribute to nursing


because it seeks to:

a. include new modalities of care


b. resolve a clinical problem
c. clarify an ambiguous modality of care
d. enhance client care

9. Martha does review of related literature for the purpose of:

a. determine statistical treatment of data research


b. gathering data about what is already known or unknown
c. to identify if problem can be replicated
d. answering the research question

10. Client’s rights should be protected when doing research using


human subjects. Martha identifies these rights as follows EXCEPT:

a. right of self-determination
b. right to compensation
c. right of privacy
d. right not to be harmed

Situation 3 - Richard has a nursing diagnosis of ineffective airway


clearance related to excessive secretions and is at risk for infection
because of retained secretions. Part of Nurse Mario's nursing care plan
is to loosen and remove excessive secretions in the airway,

11. Mario listens to Richard's bilateral sounds and finds that


congestion is in the upper lobes of the lungs. The appropriate
position to drain the anterior and posterior apical segments of
the lungs when Mario does percussion would be:

a. Client lying on his back then flat on his abdomen on Trendelenburg


position
b. Client seated upright in bed or on a chair then leaning forward in
sitting position then flat on his back and on his abdomen
c. Client lying flat on his back and then flat on his abdomen
d. Client lying on his right then left side on Trendelenburg position

12. When documenting outcome of Richard's treatment Mario should


include the following in his recording EXCEPT:

a. Color, amount and consistent of sputum


b. Character of breath sounds and respirator/rate before and after
procedure
c. Amount of fluid intake of client before and after the procedure
d. Significant changes in vital signs

13. When assessing Richard for chest percussion or chest vibration


and postural drainage Mario would focus on the following EXCEPT:

a. Amount of food and fluid taken during the last meal before treatment
b. Respiratory rate, breath sounds and location of congestion
c. Teaching the client's relatives to perform 'the procedure
d. Doctor's order regarding position restriction and client's tolerance
for lying flat

14. Mario prepares Richard for postural drainage and percussion.


Which of the flowing is a special consideration when doing the
procedure?

a. Respiratory rate of 16 to 20 per minute


b. Client can tolerate sitting and lying position
c. Client has no signs of infection
d. Time of fast food and fluid intake of the client

15. The purpose of chest percussion and vibration is to loosen


secretions in the lungs. The difference between the procedure is;

a. Percussion uses only one hand white vibration uses both hands
b. Percussion delivers cushioned blows to the chest with cupped palms
while gently shakes secretion loose on the exhalation cycle
c. In both percussion and vibration the hands are on top of each other
and hand action is in tune with client's breath rhythm
d. Percussion slaps the chest to loosen secretions while vibration
shakes the secretions along with the inhalation of air

Situation 4 - A 61 year old man, Mr. Regalado, is admitted to the


private ward for observation; after complaints of severe chest pain. You
are assigned to take care of the client.

16. When doing an initial assessment, the best way for you to
identify the client’s priority problem is to:

a. Interview the client for chief complaints and other symptoms


b. Talk to the relatives to gather data about history of illness
c. Do auscultation to check for chest congestion
d. Do a physical examination white asking the client relevant questions

17. Upon establishing Mr. Regalado's nursing needs, the next


nursing approach would be to:

a. introduce the client to the ward staff to put the client and family
at ease
b. Give client and relatives a brief tour of the physical set up the
unit
c. Take his vital signs for a baseline assessment
d. Establish priority needs and implement appropriate interventions

18. Mr. Regalado says he has "trouble going to sleep". In order to


plan your nursing intervention you will.

a. Observe his sleeping patterns in the next few days


b. Ask him what he means by this statement
c. Check his physical environment to decrease noise level
d. Take his blood pressure before sleeping and upon waking up

19. Mr. Regalado's lower extremities are swollen and shiny. He has
pitting pedal edema. When taking care of Mr. Regalado, which of
the following intervention would be the most appropriate immediate
nursing approach.

a. Moisturize lower extremities to prevent skin irritation


b. Measure fluid intake and output to decrease edema
c. Elevate lower extremities for postural drainage
d. Provide the client a list of food low in sodium

20. Mr. Regalado will be discharged from your unit within the
hour. Nursing actions when preparing a client for discharge
include all EXCEPT:

a. Making a final physical assessment before client leaves the hospital


b. Giving instructions about his medication regimen
c. Walking the client to the hospital exit to ensure his safety
d. Proper recording of pertinent data

Situation 5 - Nancy, mother of 2 young kids. 36 years old, had a


mammogram and was told that she has breast cysts and that she may need
surgery. This causes her anxiety as shown by increase in her pulse and
respiratory rate, sweating and feelings of tension.

21. Considering her level of anxiety, the nurse can best assist
Nancy by:

a. Giving her activities to divert her attention


b. Giving detailed explanations about the treatments she will undergo
c. Preparing her and her family in case surgery is not successful
d. Giving her clear but brief information at the level of her
understanding

22. Nancy blames God for her situation. She is easily provoked to
tears and wants to be left alone, refusing to eat or talk to her
family. A religious person before, she now refuses to pray or go
to church stating that God has abandoned her. The nurse
understands that Nancy is grieving for her self and is in the
stage of:

a. bargaining
b. denial
c. anger
d. acceptance

23. The nurse visits Nancy and prods her to eat her food. Nancy
replies "what's the use? My time is running out. The nurse's best
response would be:

a. "The doctor ordered full diet for you so that you will be strong for
surgery."
b. "I understand how you fee! but you have 1o try for your children's
sake."
c. "Have you told your, doctor how you feel? Are you changing your mind)
about surgery?"
d. "You sound like you are giving up."

24. The nurse feels sad about Nancy's illness and tells her head
nurse during the end of shift endorsement that "it's unfair for
Nancy to have cancer when she is still so young and with two
kinds. The best response of the head nurse would be:

a. Advise the nurse to "be strong and learn to control her feelings"
b. Assign the nurse to another client to avoid sympathy for the client
c. Reassure the nurse that the client has hope if she goes through all
statements prescribed for her
c. Ask the other nurses what they feel about the patient to find out if
they share the same feelings

25. Realizing that she feels angry about Nancy's condition, the
nurse Seams that being self-aware is a conscious process that she
should do in any situation like this because:

a. This is a necessary part of the nurse -client relationship process


b. The nurse is a role model for the client and should be strong
C. How the nurse thinks and feels affect her actions towards her client
and her work
d. The nurse has to be therapeutic at all times and should not be
affected

Situation 6 – Mrs. Seva, 32 years old, asks you about possible problems
regarding her elimination now that she is in the menopausal stage.

26. Instruction on health promotion regarding urinary elimination


is important. Which would you include?

a. Hold urine, as long as she can before emptying the bladder to


strengthen her sphincters muscles
b. If burning sensation is experienced while voiding, drink pineapple-
juice
c. After urination, wipe from anal area up towards the pubis
d. Jell client to empty the bladder at each voiding

27. Mrs. Seva also tells the nurse that she is often constipated.
Because she is aging, what physical changes predispose her to
constipation?

a. inhibition of the parasympathetic reflex


b. weakness of sphincter muscles of the anus
c. loss of tone of the smooth muscles of the color
d. decreased ability to absorb fluids in the lower intestines

28. The nurse understands that one of these factors contributes to


constipation:
a. excessive exercise
b. high fiber diet
c. no regular tine for defecation daily
d. prolonged use of laxatives

29. Mrs. Seva talks about rear of being incontinent due to a prior
experience of dribbling urine when laughing or sneezing and when
she has a full bladder. Your most appropriate .instruction would
be to:

a. tell client to drink less fluids to avoid accidents


b. instruct client to start wearing thin adult diapers
c. ask the client to bring change of underwear "just in case"
d. teach client pelvic exercise to strengthen perineal muscles

30. Mrs. Seva asked for instructions for skin care for her mother
who has urinary incontinence and is almost always in bed. Your
instruction would focus on prevention of skin irritation and
breakdown by

a. Using thick diapers to absorb urine well


b. Drying the skin with baby powder to prevent or mask the smell of
ammonia
c. Thorough washing, rising and during of skin area that get wet with
urine
d. Making sure that linen are smooth and dry at all times

Situation 7 - Using Maslow's need theory, Airway, Breathing and


Circulation are the physiological needs vital to life. The nurse's
knowledge and ability to identify and immediately intervene to meet
these needs is important to save lives.

31. Which of these clients has a problem with the transport of


oxygen from the lungs to the tissues:

a. Carol with a tumor in the brain


b. Theresa with anemia
c. Sonny Boy with a fracture in the femur
d. Brigette with diarrhea

32. You noted from the lab exams in the chart of Mr. Santos that
he has reduced oxygen in the blood.
This condition is called:

a. Cyanosis
b. Hypoxia
c. Hypoxemia
d. Anemia

33. You will nasopharyngeal suctioning Mr. Abad. Your guide for
the length of insertion of the tubing for an adult would be:

a. tip of the nose to the base of the .neck


b. the distance from the tip of the nose to the middle of the cheek
c. the distance from the tip of the nose to the tip of the ear lobe
d. eight to ten inches

34. While doing nasopharyngeal suctioning on .Mr. Abad, the nurse


can avoid trauma to the area by:

a. Apply suction for at least 20-30 seconds each time to ensure that all
secretions are removed
b. Using gloves to prevent introduction of pathogens to the respiratory
system
c. Applying no suction while inserting the catheter
d. Rotating catheter as it is inserted with gentle suction

35. Myrna has difficulty breathing when on her back and must sit
upright in bed to breath, effectively and comfortably. The nurse
documents this condition as:

a. Apnea
b. Orthopnea
c. Dyspnea
d. Tachypnea

Situation 8 - You are assigned to screen for hypertension: Your task is


to take blood pressure readings and you are informed about avoiding the
common mistakes in BP taking that lead to 'false or inaccurate blood
pressure readings.

36. When taking blood pressure reading the cuff should be:

a. deflated fully then immediately start second reading for same client
b deflated quickly after inflating up to 180 mmHg
c. large enough to wrap around upper arm of the adult client 1 cm above
brachial artery
d. inflated to 30 mmHg above the estimated systolic BP based on
palpation of radial or bronchial artery

37. Chronic Obstructive Pulmonary Disease (COPD) in one of the


leading causes of death worldwide and is a preventable disease.
The primary cause of COPD is:
a. tobacco hack
b. bronchitis
c. asthma
d. cigarette smoking

38. In your health education class for clients with diabetes you
teach, them the areas, for control . Diabetes which include all
EXCEPT:

a. regular physical activity


b. thorough knowledge of foot care
c. prevention nutrition
d. proper nutrition

39. You teach your clients the difference between, Type I (IDDM)
and Type II (NDDM) Diabetes. Which of the following is true?

a. both types diabetes mellitus clients are all prone to developing


ketosis
b. Type II (NIDDM) is more common and is also preventable compared to
Type I (IDDM) diabetes which is genetic in etiology
c. Type I (IDDM) is characterized by fasting hyperglycemia
d. Type II (IDDM) is characterized by abnormal immune response

40. Lifestyle-related diseases in general share areas common risk


factors. These are the following except
a. physical activity
b. smoking
c. genetics
d. nutrition

Situation 9 - Nurse Rivera witnesses a vehicular accident near the


hospital where she works. She decides to get involved and help the
victims of the accident.

41. Her priority nursing action would be to:

a. Assess damage to property


b. Assist in the police investigation since she is a witness
c. Report the incident immediately to the local police authorities
d. Assess the extent of injuries incurred by the victims, of the
accident

42. Priority attention should be given to which of these clients?

a. Linda who shows severe anxiety due to trauma of the accident


b. Ryan who has chest injury, is pate and with difficulty of breathing
c. Noel who has lacerations on the arms with mild-bleeding
c. Andy whose left ankle swelled and has some abrasions

43. In the emergency room, Nurse Rivera is assigned to attend to


the client with .lacerations on the arms, while assessing the
extent of the wound the nurse observes that the wound is now
starting to bleed profusely. The most immediate nursing action
would be to:

a. Apply antiseptic to prevent infection


b. Clean the wound vigorously of contaminants
c. Control and. reduce bleeding of the wound
d. Bandage the wound and elevate the arm

44. The nurse applies pressure dressing on the bleeding site. This
intervention is done to:

a. Reduce the need to change dressing frequently


b. Allow the pus to surface faster
c. Protect the wound from micro organisms in the air
d. Promote hemostasis

45. After the treatment, the client is sent home and asked to come
back for follow-up care. Your responsibilities when the client is
to be discharged include the following EXCEPT:

a. Encouraging the client to go to the, outpatient clinic for follow up


care
b. Accurate recording, of treatment done and instructions given to
client
c. Instructing the client to see you after discharge for further
assistance
d. Providing instructions regarding wound care

Situation 10 - While working in the clinic, a new client, Geline, 35


years old, arrives for her doctor's appointment. As the clinic nurse,
you are to assist the client fiil up forms, gather data and make an
assessment.

46. The nurse purpose of your initial nursing interview is to:

a. Record pertinent information in the client chart for health team to


read
b Assist the client find solutions to her health concerns
c. Understand her lifestyle, health needs and possible problems to
develop a plan of care
d. Make nursing diagnoses for identified health problems
47. While interviewing Geline, she starts to moan and doubles up
in pain, She tells you that this pain occurs about an hour after
taking black coffee without breakfast for a few weeks now. You
will record this as follows:

a. Claims to have abdominal pains after intake of coffee unrelieved by


analgesics
b. After drinking coffee, the client experienced severe abdominal pain
c. Client complained of intermittent abdominal pain an hour after
drinking coffee
d. Client reported abdominal pain an hour after drinking black coffee
for three weeks now

48. Geline tells you that she drinks black coffee frequently
within the day to "have energy and be wide awake" and she eats
nothing for breakfast and eats strictly vegetable salads for lunch
and dinner to lose weight. She has lost weight during the past two
weeks, in planning a healthy balanced diet with Geline, you will:

a. Start her off with a cleansing diet to free her body of toxins then
change to a vegetarian, diet and drink plenty of fluids
b. Plan a high protein, diet; low carbohydrate diet for her considering
her favorite food
c. Instruct her to attend classes in nutrition to find food rich in
complex carbohydrates to maintain daily high energy level
d. Discuss with her the importance of eating a variety of food from the
major food groups with plenty of fluids

49. Geline tells you that she drinks 4-5 cups of black coffee and
diet cola drinks. She also smokes up to a pack of cigarettes
daily. She confesses that she is in her 2nd month of pregnancy but
she does not want to become fat that is why she limits her food
intake. You warn or caution her about which of the following?

a. Caffeine products affect the central nervous system and may cause the
mother to have a "nervous breakdown"
b. Malnutrition and its possible effects on growth and development
problems in the unborn fetus
c. Caffeine causes a stimulant effect on both the mother and the baby
d. Studies show conclusively that caffeine causes mental retardation

50. Your health education plan for Geline stresses proper diet for
a pregnant woman and the prevention of non-communicable diseases
that are influenced by her lifestyle these include of the
following EXCEPT:
a. Cardiovascular diseases
b. Cancer
c. Diabetes Mellitus
d. Osteoporosis

Situation 11 - Management of nurse practitioners is done by qualified


nursing leaders who have had clinical experience and management
experience.

51. An example of a management function of a nurse is:

a. Teaching patient do breathing and coughing exercises


b. Preparing for a surprise party for a client
c. Performing nursing procedures for clients
d. Directing and evaluating the staff nurses

52. Your head nurse in the unit believes that the staff nurses are
not capable of decision making so she makes the decisions for
everyone without consulting anybody. This type of leadership is:

a. Laissez faire leadership


b. Democratic leadership
c. Autocratic leadership
d. Managerial leadership

53. When the head nurse in your ward plots and approves your work
schedules and directs your work, she is demonstrating:

a. Responsibility
b. Delegation
c. Accountability
d. Authority

54. The following tasks can be safely delegated' by a nurse to a


non-nurse health worker EXCEPT:

a. Transfer a client from bed to chair


b. Change IV infusions
c. Irrigation of a nasogastric tube
d. Take vital signs

55. You made a mistake in giving the medicine to the wrong client
You notify the client’s doctor and write an incident report. You
are demonstrating:

a. Responsibility
b. Accountability
c. Authority
d. Autocracy

Situation 12 – Mr. Dizon, 84 years old, is brought to the .Emergency


Room for complaint of hypertension flushed face, severe headache, and
nausea. You are doing the initial assessment of vital signs.

56. You are to measure the client’s initial blood pressure


reading by doing all of the following EXCEPT:

a. Take the blood pressure reading on both arms for comparison


b. Listen to and identify the phases of Korotkoff’s sounds
c. Pump the cuff up to around 50 mmHg above the point where the pulse is
obliterated
d. Observe procedures for infection control

57. A pulse oximeter is attached to Mr. Dizon’s finger to:

a. Determine if the client’s hemoglobin level is low and if he needs


blood transfusion
b. Check level of client’s tissue perfusion
c. Measure the efficacy of the client’s anti hypertensive medications
d. Detect oxygen saturation of arterial blood before symptoms of
hypoxemia develops

58. After a few hours in the Emergency Room, Mr. Dizon is admitted
to the ward with an order of hourly monitoring of blood pressure.
The nurse finds that the cuff is too narrow and this will cause
the blood pressure reading to be:

a. Inconsistent
b. low systolic and high diastolic pressure
c. higher than what the reading should be
d. lower than what the reading should be

59. Through the client’s health history, you gather that Mr.
Dizon smokes and drinks coffee. When taking the blood pressure of
a client who recently smoked or drank coffee, how long should be
the nurse wait before taking the client’s blood pressure for
accurate reading?

a. 15 minutes
b. 30 minutes
c. 1 hour
d. 5 minutes

60. While the client has the pulse oximeter on his fingertip, you
notice that the sunlight is shining on .the area where the
oximeter is. Your action will be to:

a. Set and turn on the alarm of the oximeter


b. Do nothing since there is no identified problem
c. Cover the fingertip sensor with a towel or bedsheet
d. Change the location of the sensor every four hours

Situation 13 - The nurse's understanding of ethico-legal


responsibilities will guide his/her nursing practice.

61. The principles that .govern right and proper conducts of a


person regarding life, biology and the health professions is
referred to as:

a. Morality
b. Religion
c. Values
d. Bioethics

62. The purpose of having nurses’ code of ethics is:

a. Delineate the scope and areas of nursing practice


b. Identify nursing action recommended for specific healthcare
situations
c. To help the public understand professional conduct, expected of
nurses
d. To define the roles and functions of the health care giver, nurses,
clients

63. The most important nursing responsibility where ethical


situations emerge in patient care is to:

a. Act only when advised that the action is ethically sound


b. Not take sides remain neutral and fair
c. Assume that ethical questions are the responsibility: of the health
team
d. Be accountable for his or her own actions

64. You inform the patient about his rights which include the
following EXCEPT:

a. Right to expect reasonable continuity of care


b. Right to consent to or decline to participate in research studies or
experiments
c. Right to obtain information about another patient
d. Right to expect that the records about his care will be treated as
confidential

65. The principle states that a person has unconditional worth and
has the capacity to determine his own destiny.

a. Bioethics
b. Justice
c. Fidelity
d. Autonomy

Situation 14 – Your director of nursing wants to improve the quality of


health care offered in the hospital. As a staff nurse in that hospital
you know that this entails quality assurance programs.

66. The following mechanisms can be utilized as part of the


quality assessment program of your hospital EXCEPT:

a. Patient satisfaction surveys provided


b. Peer review clinical records of care of client
c. RO of the Nursing Intervention Classification
d.

67. The nurse of the Standards of Nursing Practice is important in


the hospital. Which of the following statements best describes
what it is?

a. These are statements that describe the maximum or highest level of


acceptable performance in nursing practice.
b. It refers to the scope of nursing as defined in Republic Act 9173
c. It is a license issued by the Professional Regulation Commission to
protect the public from substandard nursing practice.
d. The Standards of care includes the various steps of the nursing
process and the standards of professional performance.

68. You are taking care of critically ill client and the doctor in
charge calls to order a DNR (do not resuscitate) for the client.
Which of the following is the appropriate action when getting DNR
order over the phone?

a. Have the registered nurse, family spokesperson, nurse supervisor and


doctor sign
b. Have two nurses validate the phone order, both nurses sign the order
and the doctor should sign his order within 24 hours.
c. Have the registered nurse, family and doctor sign the order
d. Have 1 nurse take the order and sign it and have the doctor sign it
within 24 hours
69. To ensure the client safety before starting blood transfusion
the following are needed before the procedure can be done EXCEPT:

a. take baseline vital signs


b. blood should be warmed to room temperature for 30 minutes before
blood transfusion is administered
c. have two nurses verify client identification, blood type, unit number
and expiration date of blood
d. get a consent signed for blood transfusion

70. Part of standards of care has to do with the use of


restraints. Which of the following statements is NOT true?

a. Doctor’s order for restraints should be signed within 24 hours


b. Remove and reapply restraints every two hours
c. Check client’s pulse, blood pressure and circulation every four
hours
d. Offer food and toileting every two hours

Situation 15 – During the NUTRITION EDUCATION class discussion a 58


year old man, Mr. Bruno shows increased interest.

71. Mr. Bruno asks what the "normal" allowable salt intake is.
Your best response to Mr. Bruno is:

a. 1 tsp of salt/day with iodine and sprinkle of MSG


b. 5 gms per day or 1 tsp of table salt/day
c. 1 tbsp of salt/day with some patis and toyo
d. 1 tsp of salt/day but not patis or toyo

72. Your instructions to reduce or limit salt intake include all


the following EXCEPT:
a. eat natural food with little or no salt added
b. limit use of table salt and use condiments instead
c. use herbs and spices
d. limit intake of preserved or processed food

73. Teaching strategies and approaches when giving nutrition


education is influenced by age, sex and immediate concerns of the
group. Your presentation for a group of young mothers would be
best if you focus on:

a. diets limited in salt and fat


b. harmful effect on drugs and alcohol intake
c. commercial preparation of dishes
d. cooking demonstration and meal planning
74. Cancer cure is dependent on

a. use of alternative methods of healing


b. watching out for warning signs of cancer
c. proficiency in doing breast self-examination
d. early detection and prompt treatment

75. The role of the health worker in health education is to:

a. report incidence of non-communicable disease to community health


center
b. educate as many people about warning signs of non-communicable
diseases
c. focus on smoking cessation projects
d. monitor clients with hypertension

Situation 16 – You are assigned to take care of 10 patients during the


morning shift. The endorsement includes the IV infusion and medications
for these clients.

76. Mr. Felipe, 36 years old is to be given 2700ml of D5RL to


infuse for 18 hours starting at 8am. At what rate should the IV
fluid be flowing hourly?

a. 100 ml/hour
b. 210 ml/hour
c. 150 ml/hour
d. 90 ml/hour

77. Mr. Atienza is to receive 150mg/hour of D5W IV infusion for 12


hours for a total of 1800ml. He is also losing gastric fluid which
must be replaced every two hours. Between 8am to 10am. Mr. Atienza
has lost 250ml of gastric fluid. How much fluid should he receive
at 11am?

a. 350 ml/hour
b. 275 ml/hour
c. 400 ml/hour
d. 200 ml/hour

78. You are to apply a transdermal patch of nitroglycerin to your


client. The following important guidelines to observe EXCEPT:

a. Apply to hairlines clean are of the skin not subject to much


wrinkling
b. Patches may be applied to distal part of the extremities like forearm
c. Change application and site regularly to prevent irritation of the
skin
d. Wear gloves to avoid any medication of your hand

79. You will be applying eye drops to Miss Romualdez. After


checking all the necessary information and cleaning the affected
eyelid and eyelashes you administer the ophthalmic drops by
instilling the eye drops.

a. directly onto the cornea


b. pressing on the lacrimal duct
c. into the outer third of the lower conjunctival sac
d. from the inner canthus going towards the side of the eye

80. When applying eye ointment, the following guidelines apply


EXCEPT:

a. squeeze about 2 cm of ointment and gently close but not squeeze eye
b. apply ointment from the inner canthus going outward of the affected
eye
c. discard the first bead of the eye ointment before application because
the tube likely to expel more than desired amount of ointment
d. hold the tube above the conjunctival sac do not let tip touch the
conjuctiva

Situation 17 – The staff nurse supervisor request all the staff nurses
to “brainstorm” and learn ways to instruct diabetic clients on self-
administration of insulin. She wants to ensure that there are nurses
available daily to do health education classess.

81. The plan of the nurse supervisor is an example of

a. in service education process


b. efficient management of human resources
c. increasing human resources
d. primary prevention

82. When Mrs. Guevarra, a nurse, delegates aspects of the clients


care to the nurse-aide who is an unlicensed staff, Mrs. Guevarra.

a. makes the assignment to teach the staff member


b. is assigning the responsibility to the aide but not the
accountability for those tasks
c. does not have to supervise or evaluate the aide
d. most know how to perform task delegated

83. Connie, the-new nurse, appears tired and sluggish and lacks
the enthusiasms she give six weeks ago when she started the job.
The nurse supervisor should:

a. empathize with the nurse and listen to her


b. tell her to take the day off
c. discuss how she is adjusting to her new job
d. ask about her family life

84. Process of formal negotiations of working conditions between a


group of registered nurses and employer is:

a. grievance
b. arbitration
c. collective bargaining
d. strike

85. You are attending a certification program on cardiopulmonary


resuscitation (CPR) offered and required by the hospital employing
you. This is;

a. professional course towards credits


b. in-service education
c. advance training
d. continuing education

Situation 18 - There are various developments in health education that


the nurse should know about.

86. The provision of health information in the rural areas


nationwide through television and radio programs and video
conferencing is referred to as:

a. Community health program


b. Telehealth program
c. Wellness program
d. Red cross program

87. A nearby community provides blood pressure screening, height


and weight measurement smoking cessation classes and aerobics
class services. This type of program is referred to as:

a. outreach program
b. hospital extension program
c. barangay health center
d. wellness center

88. Part of teaching client in health promotion is responsibility


for one’s health. When Danica states she need to improve her
nutritional status this means:

a. Goals and interventions to be followed by client are based on nurse's


priorities
b. Goals and intervention developed by nurse and client should be
approved by the doctor
c. Nurse will decide goals and, interventions needed to meet client
goals
d. Client will decide the goals and interventions required to meet her
goals

89. Nurse Beatrice is providing tertiary prevention to Mrs. De


Villa. An example of tertiary provestion is:

a. Marriage counseling
b. Self-examination for breast cancer
c. Identifying complication of diabetes
d. Poison, control

90. Mrs. Ostrea has a schedule for Pap Smear. She has a strong
family history of cervical cancer. This is an example of:

a. tertiary prevention
b. secondary prevention
c. health screening
d. primary prevention

Situation: 19 - Ronnie has a vehicular accident where he sustained


injury to his left ankle. In the Emergency Room, you notice how anxious
he looks.

91. You establish rapport with him and to reduce his anxiety you
initially

a. Take him to the radiology, section for X-ray of affected extremity


b. Identify yourself and state your purpose in being with the client
c. Talk to the physician for an order of Valium
d. Do inspection and palpation to check extent of his injuries

92. While doing your assessment, Ronnie asks you "Do I have a
fracture? I don't want to have a cast.” The most appropriate
nursing response would be:

a. "You have to have an X-ray first to know if you have a fracture."


b. "Why do you; sound so scared? It is just a cast and it's not painful"
c. "You seem to be concerned about being in a cast."
d. "Based on my assessment, there doesn’t seem to be a fracture."
ANSWER KEY
1. C
2. B
3. D
4. D
5. B
6. B
7. C
8. D
9. B
10. B
11. B
12. C
13. C
14. D
15. A
16. A
17. C
18. B
19. A
20. C
21. D
22. C
23. D
24. D
25. C
26. D
27. C
28. D
29. D
30. C
31. B
32. C
33. C
34. C
35. B
36. D
37. D
38. B
39. B
40. C
41. D
42. B
43. D
44. D
45. C
46. C
47. D
48. D
49. B
50. D
51. D
52. C
53. D
54. B
55. B
56. C
57. D
58. C
59. B
60. C
61. D
62. C
63. D
64. C
65. D
66. D
67. A
68. D
69. D
70. C
71. B
72. B
73. D
74. D
75. B
76. C
77. -
78. B
79. B
80. C
81. C
82. B
83. C
84. C
85. B
86. B
87. A
88. D
89. C
90. B
91. B
92. C

PREBOARD EXAM 2

COMMUNITY HEALTH NURSING AND CARE OF THE MOTHER AND CHILD

Situation 1 - Nurse Minette is an independent Nurse Practitioner


following-up referred clients in their respective homes. Here she
handles a case of POSTPARTIAL MOTHER AND FAMILY focusing on HOME CARE.

1. Nurse Minette needs to schedule a first home visit to OB client


Leah. When is a first home-care visit typically made?

a. Within 4 days after discharge


b. Within 24 hours after discharge
c. Within 1 hour after discharge
d. Within 1 week of discharge

2. Leah is developing constipation from being on bed rest. What


measures would you suggest she take to help prevent this?

a. Eat more frequent small meals instead of three large one daily
b. Walk for at least half an hour daily to stimulate peristalsis
c. Drink more milk, increased calcium intake prevents constipation
d. Drink eight full glasses of fluid such as water daily

3. If you were Minette, which of the following actions, would


alert you that a new mother is entering a postpartial at taking-
hold phase?

a. She urges the baby to stay awake so that she can breast-feed him in
her
b. She tells you she was in a lot of pain all during labor
c. She says that she has not selected a name fir the baby as yet
d. She sleeps as if exhausted from the effort of labor

4. At 6-week postpartum visit what should this postpartial


mother's fundic height be?

a. Inverted and palpable at the cervix


b. Six fingerbreadths below the umbilicus
c. No longer palpable on her abdomen
d. One centimeter above the symphysis pubis

5. This postpartal mother wants to loose the weight she gained in


pregnancy, so she is reluctant to increase her calorin intake for
breast-feeding. By how much should a lactating mother increase her
caloric intake during the first 6 months after birth?

a. 350 kcal/day
b. 5CO kcal/day
c. 200 kcal/day
d. 1,000 kcal/day

Situation 2 - As the CPES is applicable for all professional nurse, the


professional growth and development of Nurses with specialties shall be
addressed by a Specialty Certification Council.
The following questions apply to these special groups of nurses.

6. Which of the following serves as the legal basis and statute


authority for the Board of nursing to promulgate measures to
effect the creation of a Specialty Certification Council and
promulgate professional development programs for this group of
nurse-professionals?

a. R.A. 7610
b. R.A. 223
c. R.A. 9173
d. R.A. 7164

7. By force of law, therefore, the PRC-Board of Nursing released


Resolution No. 14 Series of the entitled: "Adoption of a Nursing
Specialty Certification Program and Creation of Nursing Specialty
Certification Council." This rule-making power is called:

a. Quasi-Judicial Power
b. Regulatory Power
c. Quasi/Legislative Power
d. Executive/Promulgation Power

8. Under the PRC-Board of Nursing Resolution promulgating the


adoption of a Nursing Specialty-Certification Program and Council,
which two (2) of the following serves as the strongest for its
enforcement?
(a) Advances made in science aid technology have provided the climate
for specialization in almost all aspects of human endeavor and
(b) As necessary consequence, there has emerged a new concept known as
globalization which seeks to remove barriers in trade, .industry and
services imposed by the national laws of countries all over the world;
and
(c) Awareness of this development should impel the nursing sector to
prepare our people in the services sector to meet .the above challenges;
and
(d) Current trends of specialization in nursing practice recognized by;
the International Council of Nurses (ICN) of which the Philippines is a
member for the benefit of the Filipino in terms of deepening and
refining nursing practice and enhancing the quality of nursing care.

a. b & c are strong justification


b. a & b are strong justification
c. a & c are strong justification
d. a & d are strong justification

9. Which of the following is NOT a correct statement as regards


Specialty Certification?

a. The Board of Nursing intended to create the Nursing Specialty


Certification Program as a means of perpetuating the creation of an
elite force of Filipino Nurse Professionals
b. The Board of Nursing shall oversee the administration of the NSCP
through the various Nursing Specialty Boards which will eventually, be
created
c. The Board of Nursing at the time exercised their powers under R.A.
7164 in order to adopt the creation of the Nursing Specialty
Certification /council and Program
d. The Board of Nursing consulted nursing leaders of national nursing
associations and other concerned nursing groups which later decided to
ask a special group of nurses of .the program for nursing specialty
certification

10. The NSCC was created for the purpose of implementing the
Nursing Specialty policy under the direct supervision and
stewardship of the Board of Nursing. Who shall comprise the NSCC?

a. A Chairperson who is the current President of the APO a member


from .the Academe, and the last member coming from the Regulatory Board
b. The Chairperson and members of the Regulatory Board ipso facto acts
as the CPE Council
c. A Chairperson, chosen from among the Regulatory Board Members, a Vice
Chairperson appointed by the BON at-large; two other members also chosen
at-large; and one representing the consumer group
d. A Chairperson who is the President of the Association from the
Academe; a member from the Regulatory Board, and the last member coming
from the APO

Situation 3 - Nurse Anna is a new BSEN graduate and has just passed her
Licensure Examination for Nurses in the Philippines. She has likewise
been hired as a new Community Health Nurse in one of the Rural Health
Units in their City, which of the following conditions may be acceptable
TRUTHS applied to Community Health Nursing Practice.
11. Which of the following is the primary focus of community
health nursing practice?

a. Cure of illnesses
b. Prevention of illness
c. Rehabilitation back to health
d. Promotion of health

12. In community health nursing, which of the following is our


unit of service as nurses?

a. The Community
b. The Extended Members of every family
c. The individual members of the Barangay
d. The Family

13. A very important part of the Community Health Nursing


Assessment Process includes

a. the application of professional judgment in estimating importance of


facts to family and community
b. evaluation structures arid qualifications of health center team
c. coordination with other sectors in relation to health concerns
d. carrying out nursing procedures as per plan of action

14. In community health nursing it is important to take into


account the family health with an equally important need to
perform ocular inspection of the areas activities which are
powerful elements of:

a. evaluation
b. assessment
c. implementation
d. planning

15. The initial step in the PLANNING process in order to engage in


any nursing project or parties at the community level involves:

a. goal-setting
b. monitoring
c. evaluation of data
d. provision of data

Situation 4 - Please continue responding as a professional nurse in


these other health situations through the following questions.
16. Transmission of HIV from an infected individual to another
person occurs:

a. Most frequency in nurses with needlesticks


b. Only if there is a large viral load in the blood
c. Most commonly as a result of sexual contact
d. In all infants born to women with HIV infection

17. The medical record of a client reveals a condition in which


the fetus cannot pass through the maternal pelvis. The nurse
interprets this as:

a. Contracted pelvis
b. Maternal disproportion
c. Cervical insufficiency
d. Fetopelvic disproportion

18. The nurse would anticipate a cesarean birth for a client who
has which infection present at the onset of labor?

a. Herpes simplex virus


b. Human papilloma virus
c. Hepatitis
d. Toxoplasmosia

19. After a vaginal examination, the nurse»e determines that the


client's fetus is in an occiput posterior position. The nurse
would anticipate that the client will have:

a. A precipitous birth
b. Intense back pain
c. Frequent leg cramps
d. Nausea and vomiting

20. The rationales for using a prostaglandin gel for a client


prior to the induction of labor is to:

a. Soften and efface the cervix


b. Numb cervical' pain receptors
c. Prevent cervical lacerations
d. Stimulate uterine contractions

Situation 5 - Nurse Lorena is a Family Planning and Infertility Nurse


Specialist and currently attends to FAMILY PANNING CLIENTS AND INFERTILE
COUPLES. The following conditions pertain to meeting the nursing of this
particular population group.
21. Dina, 17 years old, asks you how a tubal ligation prevents
pregnancy. Which would be the best answer?

a. Prostaglandins released from the cut fallopian tubes can kill sperm
b. Sperm cannot enter the uterus, because the cervical entrance is
blocked
c. Sperm can no longer reach the ova, because the fallopian tubes are
blocked
d. The ovary no longer releases ova, as there is no where for them to go

22. The Dators are a couple undergoing testing for infertility.


Infertility is said to exist when:

a. a woman has no uterus


b. a woman has no children
c. a couple has been trying to conceive for 1 year
d. a couple has wanted a child for 6 months

23. Another client names Lilia is diagnosed as having


endometriosis. This condition interferes with the fertility
because:

a. endometrial implants can block the fallopian tubes


b. the uterine cervix becomes inflamed and swollen
c. ovaries stop producing adequate estrogen
d. pressure on the pituitary leads to decreased FSH levels

24. Lilia is scheduled to have a hysterosalpingogram. Which of the


following, instructions would you give her regarding this
procedure?

a. She will not be able to conceive for 3 months after the procedure
b. The sonogram of the uterus will reveal any tumors present
c. Many women experience mild bleeding as an after effect
d. She may feel some cramping when the dye is inserted

25. Lilia's cousin on the other hand, knowing nurse Lorena's


specialization asks what artificial insemination by donor entails.
Which would be your best answer if you were Nurse Lorena?

a. Donor sperm are introduced vaginally into the uterus or cervix


b. Donor sperm are injected intra-abdominally into each ovary
c. Artificial sperm are injected vaginally to test tubal patency
d. The husband's sperm is administered intravenously weekly

Situation 6 - There are other important basic knowledge in the


performance of our task as Community Health Nurse in relation to
IMMUNIZATION these include:

26. The correct temperature to store vaccines in a refrigerator


is:

a. between -4 deg C and +8 deg C


b. between 2 deg C and +8 deg C
c. between -8 deg C and 0 deg C
d. between -8 deg C and +8 deg C

27. Which of the following vaccines is not done by intramuscular


(IM) injection?

a. Measles vaccine
b. DPT
c. Hepa B vaccines
d. DPT

28. This vaccine content is derived from RNA recombinants:

a. Measles
b. Tetanus toxoids
c. Hepatitis B vaccines
d. DPT

29. This is the vaccine needed before a child reaches one (1) year
in order for him/her to qualify as a "fully immunized child".

a. DPT
b. Measles
c. Hepatitis B
d. BCG

30. Which of the following dose of tetanus toxoid is given to the


mother to protect her .infant from neonatal tetanus and likewise
provide 10 years protection for the mother?

a. Tetanus toxoid 3
b. Tetanus toxoid 2
c. Tetanus toxoid 1
d. Tetanus toxoid 4

Situation 7 - Records contain those, comprehensive descriptions of


patient's health conditions and needs and at the same serve as evidences
of every nurse's accountability in the, care giving process. Nursing
records normally differ from institution to, institution nonetheless
they follow similar patterns of .meeting needs for specifics, types of
information. The following pertalos to documentation/records management.

31. This special form used when the patient is admitted to the
unit. The nurse completes, the information in this records
particularly his/her .basic personal data, current illness,
previous health history, health history of the family, emotional
profile, environmental history as well as physical assessment
together with nursing diagnosis on admission. What do you call
this record?

a. Nursing Kardex
b. Nursing Health History and Assessment Worksheet
c. Medicine and Treatment Record
d. Discharge Summary

32. These, are sheets/forms which provide an efficient and time


saving way to record information that must be obtained repeatedly
at regular and/or short intervals, of .time. This does not replace
the progress notes; instead this record of information on vital
signs, intake and output, treatment, postoperative care,
postpartum care, and diabetic regimen, etc., this is used whenever
specific measurements or observations are needed to-be documented
repeatedly. What is this?

a. Nursing Kardex
b. Graphic Flow sheets
c. Discharge Summary
d. Medicine and Treatment Record

33. These records show all medications and treatment provided on a


repeated basis. What do you call this record?

a. Nursing Health History and Assessment Worksheet


b. Discharge Summary
c. Nursing Kardex
d. Medicine and Treatment Record

34. This flip-over card is usually kept in a portable file at the


Nurses Station. It has 2-parts: the activity and treatment section
and a nursing care plan section. This carries information about
basic demographic data, primary medical diagnosis, current orders
of the physician to be carried out by the nurse, written nursing
care plan, nursing orders, scheduled tests and procedures, safety
precautions in-patient care and factors related to daily living
activities/ this record is used in the charge-of-shift reports or
during the beside rounds or walking rounds. What record is this?
a. Discharge Summary
b. Medicine and Treatment Record
c. Nursing Health History and Assessment Worksheet
d. Nursing Kardex

35. Most nurses regard this as conventional recording of the date,


time and mode by which the patient leaves a healthcare unit but
this record includes importantly, directs of planning for
discharge that starts soon after the' person is admitted to a
healthcare institution, it is accepted that collaboration or
multidisciplinary involvement (of all members of the health team)
in discharge results in comprehensive care. What do you call this?

a. Discharge Summary
b. Nursing Kardex
c. Medicine and Treatment Record
d. Nursing Health History and Assessment Worksheet

Situation 8 - As Filipino Professional Nurses we must be knowledgeable,


about the Code of Ethics for Filipino Nurses and practice these by
heart. The next questions pertain to this Code of Ethics.

36. Which of the following is TRUE about the Code of Ethics of


Filipino Nurses?

a. The Philippine Nurses Association for being the accredited


professional organization was given the privilege to formulate a Code of
Ethics which the Board of Nurses promulgated
b. Code of Nurses was first formulated in 1982 published in the
Proceedings of the Third Annual Convention of the PNA House of Delegates
c. The present code utilized the Code of Good Governance for the
Professions in the Philippines
d. Certificate of Registration of registered nurses; may be revoked or
suspended for violations of any provisions of the Code of Ethics

37. Based on the Code of Ethics for Filipino Nurses, what is


regarded as the hallmark of nursing responsibility and
accountability?

a. Human rights of clients, regardless of creed and gender


b. The privilege of being a registered professional nurses
c. Health, being a fundamental right of every individual
d. Accurate documentation of actions and outcomes

38. Which of the following nurses behavior is regarded as a


violation of the Code of Ethics of Filipino Nurses?
a. A nurse withholding harmful information to the family members of a
patient
b. A nurse declining commission sent by a doctor for her referral
c. A nurse endorsing a person running for congress
d. Nurse Reviewers and/or nurse review center managers who pays a
considerable amount of cash for reviewees who would memorize items from
the Licensure exams and submit these to them after the examination

39. A nurse should be cognizant that professional programs for


specialty certification by the Board of Nursing are accredited
through the

a. Professional Regulation Commission


b. Nursing Specialty Certification Council
c. Association of Deans of Philippine Colleges of Nursing
d. Philippine Nurse Association

40. Mr. Santos, R.N. works in a nursing home, and he knows that
one of his duties is to be an advocate for his patients. Mr.
Santos knows a primary duty of an advocate is to:

a. act as the patient's legal representative


b. complete all nursing responsibilities on time
c. safeguard the well being of every patient
d. maintain the patient's right to privacy

Situation 9 - Nurse Joanna works as an OB-Gyne Nurse and attends to


several HIGH-RISK PREGNANCIES: Particularly women with preexisting of
Newly Acquired illness. The following conditions apply.

41. Bernadette is a 22-year old woman. Which condition would make


her more prone than others to developing a Candida infection
during pregnancy?

a. Her husband plays gold 6 days a week


b. She was over 35 when she became pregnant
c. She usually drinks tomato juice for breakfast
d. She has developed gestational diabetes

42. Bernadette develops a deep-vein thrombosis following an auto


accident and is prescribed heparin sub-Q. What should Joanna
educate her about in regard to this?

a. Some infants will be born with allergic symptoms to heparin


b. Her infant will be born with scattered petechiae on his trunk
c. Heparin can cause darkened skin in newborns
d. Heparin does not cross the placenta and so does not affect a fetus
43. The cousin of Bernadette with sickle-cell anemia alerted
Joanna that she may need further instruction on prenatal care.
Which statement signifies this fact?

a. I've stopped jogging so I don't risk becoming dehydrated


b. I take an iron pull every day to help grown new red blood cells
c. I am careful to drink at least eight glasses of fluid everyday
d. 1 understand why folic acid is important for red cell formation

44. Bernadette routinely takes acetylsalicylic acid (aspirin) for


arthritis. Why should she limit or discontinue this toward the end
of pregnancy?

a. Aspirin can lead to deep vein thrombosis following birth


b. Newborns develop a red rash from salicylate toxicity
c. Newborns develop withdrawal headaches from salicylates
d. Salicyates can lead to increased maternal bleeding at childbirth

45. Bernadette received a laceration on her leg from her


automotive accident. Why are lacerations of lower extremities
potentially more serious in pregnant women than others?

a. Lacerations can provoke allergic responses because of gonadothropic


hormone
b. Increased bleeding can occur from uterine pressure on leg veins
c. A woman is less able to keep the laceration clean because o f her
fatigue
d. Healing is limited during pregnancy, so these will not heal until
after birth

Situation 10 - Still in your self-managed Child Health Nursing Clinic,


your encounter these cases pertaining to the CARE OF CHILDREN WITH
PULMONARY AFFECTIONS.

46. Josie brought her 3-rnonths old child to your clinic because
of cough and colds. Which of the following is your primary action?

a. Give contrimoxazole tablet or syrup


b. Assess the patient using the chart on management of children with
cough
c. Refer to the doctor
d. Teach the mother how to count her child's bearing

47. In responding to the care concerns of children with severe


disease, referral to the hospital of the essence especially if the
child manifests which of the following?
a. Wheezing
b. Stopped bleeding
c. Fast breathing
d. Difficulty to awaken

48. Which of the following is the most important responsibility of


a nurse in the prevention of necessary deaths from pneumonia and
other severe diseases?

a. Giving of antibiotics
b. Taking of the temperature of the sick child
c. Provision of Careful Assessment
d. Weighing of the sick child

49. You were able to identify factors that lead to respiratory


problems in the community where your health facility serves. Your
primary role therefore in order to reduce morbidity due to
pneumonia is to:

a. Teach mothers how to recognize early signs and symptoms of pneumonia


b. Make home visits to sick children
c. Refer cases to hospitals
d. Seek assistance and mobilize the BHWs to have a meeting with mothers

50. Which of the following is the principal focus on the CARI


program of the Department of Health?

a. Enhancement of health team capabilities


b. Teach mothers how to detect signs and where to refer
c. Mortality reduction through early detection
d. Teach other community health workers how to assess patients

Situation 11 - You are working as a Pediatric Nurse in your own Child


Health Nursing Clinic, the following cases pertain to ASSESSMENT AND
CARE OP THE NEWBORN AT RISK conditions.

51. Theresa, a mother with a 2 year old daughter asks, "at what
are can I be able to take the blood pressure of my daughter as a
routine procedure since hypertension is common in the family?"
Your answer to this is:

a. At 2 years you may


b. As early as 1 year old
c. When she's 3- years old
d. When she's 6 years old?
52. You typically gag children to inspect the back of their
throat. When is it important NOT to solicit a gag reflex?

a. when a girl has a geographic tongue


b. when a boy has a possible inguinal hernia
c. when a child has symptoms of epiglottitis
d. when children are under 5 years of age

53. Baby John was given a drug at birth to reverse the effects of
a narcotic given to his mother in' labor. What drug is commonly
used for this?

a. Naloxone (Narcan)
b. Morphine Sulfate
c. Sodium Chloride
d. Penicillin G

54. Why are small-for-gestational-age newborns at risks for


difficulty maintaining body temperature?

a. They do not have as many fat stores as other infant’s


b. They are more active than usual so throw off covers
c. Their skin is more susceptible to conduction of cold
d. They are preterm so are born relatively small in size

55. Baby John develops hyperbilirubinemia. What is a method used


to treat hyperbilirubinemia in a newborn?

a. Keeping infants in a warm arid dark environment


b. Administration of a cardiovascular stimulant
c. Gentle exercise to stop muscle breakdown
d. Early feeding to speed passage of meconium

Situation 12 - You are the nurse in the Out-Patient-Department and


during your shift you encountered multiple children's condition. The
following questions apply.

56. You assessed a child with visible severe wasting, he has:

a. edema
b. LBM
c. kwashiorkor
d. marasmus

57. Which of the following conditions is NOT true about


contraindication to immunization?
a. do not give DPT2 or DPT3 to a child who has convulsions within 3 days
of DPT1
b. do not give BOG if the child has known hepatitis .
c. do not give OPT to a child who has recurrent convulsion or active
neurologic disease
d. do not give BCG if the child has known AIDS

58. Which of the following statements about immunization is NOT


true:

a. A child with diarrhea who is due for OPV should receive the OPV and
make extra dose on the next visit
b. There is no contraindication to immunization if the child is well
enough to go home
c. There is no contraindication to immunization if the child is well
enough to go home and a child should be immunized in the health center
before referrals are both correct
d. A child should be immunized in the center before referral

59. A child with visible severe wasting or severe palmar pallor


may be classified as:

a. moderate malnutrition/anemia
b. severe malnutrition/anemia
c. not very tow weight no anemia
d. anemia/very low weight

60. A child who has some palmar pallor can be classified as:

a. moderate anemia/normal weight


b. severe malnutrition/anemia
c. anemia/very low weight
d. not very low eight to anemia

Situation 13 - Nette, a nurse palpates the abdomen of Mrs. Medina, a


primigravida. She is unsure of the date of her last menstrual period.
Leopold's Maneuver is done. The obstetrician told mat she appears to be
20 weeks pregnant. .

61. Nette explains this because the fundus is:

a. At the level the umbilicus, and the fetal heart can be heard with a
fetoscope
b. 18 cm, and the baby is just about to move
c. is just over the symphysis, and fetal heart cannot be heard
d. 28 cm, and fetal heart can be heard with a Doppler
62. In doing Leopold's maneuver palpation which among the
following is NOT considered a good preparation?

a. The woman should lie in a supine position wither knees flexed


slightly
b. The hands of the nurse should be cold so that abdominal muscles would
contract and tighten
c. Be certain that your hands are warm (by washing them in warm water
first if necessary)
d. The woman empties her bladder before palpation

63. In her pregnancy, she experienced fatigue and drowsiness. This


probably occurs because:

a. of high blood pressure


b. she is expressing pressure
c. the fetus utilizes her glucose stores and leaves her with a Sow blood
glucose
d. of the rapid growth of the fetus

64. The nurse assesses the woman at 20 weeks gestation3 and


expects the woman to report:

a. Spotting related to fetal implantation


b. Symptoms of diabetes as human placental lactogen is released
c. Feeling fetal kicks
d. Nausea and vomiting related HCG production

65. If Mrs. Medina comes to you for check-up on June 2, her EDO is
June 11, what do you expect during assessment?

a. Fundic ht 2 fingers below xyphoid process, engaged


b. Cervix close, uneffaced, FH-midway between the umbilicus and
symphysis pubis
c. Cervix open, fundic ht. 2 fingers below xyphoid process, floating .
d. Fundic height at least at the level of the xyphoid process, engaged

Situation 14: - Please continue responding as a professional nurse in


varied health situations through the following questions.

66. Which of the following medications would the nurse expect the
physician to order for recurrent convulsive seizures of a 10-year
old child brought to your clinic?

a. Phenobarbital
b. Nifedipine
c. Butorphanol
d. Diazepam

67. RhoGAM is given to Rh-negative women to prevent maternal


sensitization from occurring. The nurse is aware that in addition
to pregnancy, Rh-negative women would also receive this medication
after which of the following?

a. Unsuccessful artificial insemination procedure


b. Blood transfusion after hemorrhage
c. Therapeutic or spontaneous abortion
d. Head injury from a car accident

68. Which of the following would the nurse include when describing
the pathophysiologv of gestational diabetes?

a. Glucose levels decrease to accommodate fetal growth


b. Hypoinsulinemia develops early in the first trimester
c. Pregnancy fosters the development of carbohydrate cravings
d. There is progressive resistance to the effects of insulin

69. When providing prenatal education to a pregnant woman with


asthma, which of the following would be important for the nurse to
do?

a. Demonstrate how to assess her blood glucose


b. Teach correct administration of subcutaneous bronchodilators
c. Ensure she seeks treatment for any acute exacerbation
d. Explain that she should avoid steroids during her pregnancy

70. Which of the following conditions would cause an insulin-


dependent diabetic client the most difficulty during her
pregnancy?

a. Rh incompatibility
b. Placenta previa
c. Hyperemesis gravidarum
d. Abruption placentae

Situation 15 - One important toot a community health nurse uses in the


conduct of his/her activities is the CHN Bag. Which of the following
BEST DESCRIBES the use of this vital facility for our practice?

71. The Community/Public Health Bag is:

a. a requirement for home visits


b. an essential and indispensable equipment of the community health
nurse
c. contains basic medications and articles used by the community health
nurse
d. a tool used by the Community health nurse is rendering effective
nursing procedure during a home visit

72. What is the rationale in the use of bag technique during home
visit?

a. It helps render effective nursing care to clients or other members of


the family
b. It saves time and effort of the nurse in the performance of nursing
procedures
c. It should minimize or prevent the spread of infection from
individuals to families
d. It should not overshadow concerns for the patient

73. Which among the following is important in the use of the bag
technique during home visit?

a. Arrangement of the bag's contents must be convenient to the nurse


b. The bag should contain all necessary supplies and equipment ready for
use
c. Be sure to thoroughly clean your bag especially when exposed to
communicable disease cases
d. Minimize if not totally prevent the spread of infection

74. This is an important procedure of the nurse during home


visits?

a. protection of the CHN bag


b. arrangement of the contents of the CHM bag
c. cleaning of the CHN bag
d. proper handwashing

75. In consideration of the steps in applying the bag technique,


which side of the paper lining of the CHN bag is considered clean
to make a non-contaminated work area?

a. The lower lip


b. The outer surface
c. The upper lip
d. The inside surface

Situation 16 - As a Community Health Nurse relating with people in


different communities, and in the implementation of health programs and
projects you experience vividly as well the varying forms of leadership
and management from the Barangay Level to the Local Government/Municipal
City Level.

76. The following statements can correctly be made about


Organization and management?

A. An organization (or company) is people. Values make people persons:


values give vitality, meaning and direction to a company. As the people
of an organization value, so the company becomes.
B. Management is the process by which administration achieves its
mission, goals, and objectives
C. Management effectiveness can be measured in terms of accomplishment
of the purpose of the organization while management efficiency is
measured in terms of the satisfaction of individual motives
D. Management principles are universal therefore one need not be
concerned about people, culture, values, traditions and human relations.

a. B and C only
b. A, B and D only
c. A and D only
d. B, A, and C only

77. Management by Filipino values advocates the consideration of


the Filipino goals trilogy according to the Filipino priority-
values which are:

a. Family goals, national goals, organizational goals


b. Organizational goats, national goals, family goals
c. National goals, organizational goals, family goals
d. Family goals, organizational goals, national goals

78. Since the advocacy for the utilization of Filipino value-


system in management has been encouraged, the Nursing sector is no
except, management needs to examine Filipino values and discover
its positive potentials and harness them to achieve:

a. Employee satisfaction
b. Organizational commits .ants, organizational objectives and employee
satisfaction
c. Employee objectives/satisfaction, commitments and organizational
objectives
d. Organizational objectives, commitments and employee
objective/satisfaction

79. The following statements can correctly be made about an


effective and efficient community or even agency managerial-
leader.
A. Considers the achievement and advancement of the organization she/he
represents as well as his people
B. Considers the recognition of individual efforts toward the
realization of organizational goals as well as the welfare of his people
C. Considers the welfare of the organization above all other
consideration by higher administration
D. Considers its own recognition by higher administration for purposes
of promotion and prestige

a. Only C and D are correct


b. A, C and D are correct
c. B, C, and D are correct
d. Only A and B are correct

80. Whether management at the community or agency level, there are


3 essential types of skills managers must have, these are:
A. Human relation skills, technical skills, and cognitive skills
B. Conceptual skills, human relation/behavioral skills, and technical
skills
C. Technical skills, budget and accounting skills, skills in fund-
raising
D. Manipulative skills, technical skills, resource management skills

a. A and D are correct


b. B is correct
c. A is correct
d. C and D are correct

Situation 17 - You are actively practicing nurse who just finished your
Graduate Studies. You earned the value of Research and would like to
utilize the knowledge and skills gained in the application of research
to Nursing service. The following questions apply to research.

81. Which type of research Inquiry investigates the issue of human


complexity (e.g. understanding the human expertise)

a. Logical position
b. Naturalistic inquiry
c. Positivism
d. Quantitative Research

82. Which of the following studies is based on quantitative


research?

a. A study examining the bereavement process in spouses of clients with


terminal cancer
b. A study exploring factors influencing weight control behavior
c. A study measuring the effects of sleep deprivation on wound healing
d. A study examining client's feelings before, during and after a bone
marrow aspiration

83. Which of the following studies is based on qualitative


research?

a. A study examining clients reactions to stress after open heart


surgery
b. A study measuring nutrition and weight, loss/gain in clients with
cancer
c. A study examining oxygen levels after endotracheal suctioning
d. A study measuring differences in blood pressure before during and
after a procedure

84. An 85 year old client in a nursing home tells a nurse, "I


signed the papers for that research study because the doctor was
so insistent and I want: him to continue taking care of me." Which
client right is being violated?

a. Right of self determination


b. Right to privacy and confidentiality
c. Right to full disclosure
d. Right not to be harmed

85. "A supposition or system of ideas that is proposed to explain


a given phenomenon," best defines:

a. a paradigm
b. a concept
c. a theory
d. a conceptual framework

Situation 18 - Nurse Michelle works with a Family Nursing Team in


Calbayog Province specifically handling a UNICEF Project for Children.
The following conditions pertain, to CARE OP THE FAMILIES PRESCHOOLERS.

86. Ronnie asks constant questions. How many does a typical 3-


year-old ask in a day's time?

a. 1,200 or more
b. Less than 50
c. 100-200
d. 300-400

87. Ronnie will need to change to a new bed because his baby
sister will need Ronnie's old crib. What measure would you suggest
that his parents take to help decrease sibling rivalry between
Ronnie and his new sister?

a. Move him to the new bed before the baby arrives


b. Explain that new sisters grow up to become best friends
c. Tell him he will have to share with the new baby
d. Ask him to get his crib ready for the new baby

88. Ronnie's parents want to know how to react to him when he


begins to masturbate while watching television. What would you
suggest?

a. They refuse to allow him to watch television


b. They schedule a health check-up for sex-related disease
c. They remind him that some activities are private
d. They give him "timeout" when this begins

89. How many words does a typical 12-month-old infant use?

a. About 12 words
b. Twenty or more words
c. About 50 words
d. Two, plus "mama" and "dada"

90. As a nurse. You reviewed infant safety procedures with Bryan's


mother. What are two of the most common types of accidents among
infants?

a. Aspiration and falls


b. Falls and auto accidents
c. Poisoning and burns
d. Drowning and homicide

Situation 19 - Among common conditions found in children especially


among poor communities are ear infection/problems. The following
questions apply.

91. A child with ear problem should be assessed for the following
EXCEPT:

a is there any fever?


b. ear discharge
c. if discharge is present for how long?
d. ear pain

92. If the child does not have ear problem, using IMCI, what
should you as the nurse do?
a. Check for ear discharge
b. Check for tender swellings, behind the ear
c. Check for ear pain
d. Go to the next question, check for malnutrition

93. An ear discharge that has been present for more than 14 days
can be classified as:

a. mastoditis
b. chronic ear infection
c. acute ear infection
d. complicated ear infection

94. An ear discharge that has been present for jess than 14 days
can be classified as:

a. chronic ear infection


b. mastoditis
c. acute ear infection
d. complicated ear infection

95. If the child has severe classification because of ear problem,


what would be the best thing that you as the nurse can do?

a. instruct mother when to return immediately


b. refer urgently
c. give an antibiotic for 5 days
d. dry the ear by wicking

Situation 20 - If a child with diarrhea registers one sign in the pink


row and one in the yellow; row in the IMCI Chart.

96. We can classify the patient as:

a. moderate dehydration
b. some dehydration
c. no dehydration
d. severe dehydration

97. The child with no dehydration needs home treatment Which of


the following is not included the rules for home treatment in this
case:

a. continue feeding the child


b. give oresol every 4 hours
c. know when to return to the health center
d. give the child extra fluids
98. A child who has had diarrhea for 14 days but has no sign of
dehydration is classified as:

a. severe persistent diarrhea


b. dysentery
c. severe dysentery b. dysentery
d. persistent diarrhea

99. If the child has sunken eyes, drinking eagerly, thirsty and
skin pinch goes back slowly, the classification would be:

a. no dehydration
b. moderate dehydration
c. some dehydration
d. severe dehydration

100. Carlo has had diarrhea for 5 days. There is no blood in the
stool, he is irritable. His eyes are sunken the nurse offers fluid
to Carlo and he drinks eagerly. When the nurse pinched the
abdomen, it goes back slowly. How will you classify Carlo’s
illness?

a. severe dehydration
b. no dehydration
c. some dehydration
d. moderate dehydration

ANSWER KEY:
1. A
2. B
3. A
4. C
5. B
6. D
7. C
8. D
9. A
10. B
11. D
12. D
13. A
14. B
15. A
16. C
17. D
18. A
19. B
20. D
21. C
22. C
23. A
24. C
25. A
26. B
27. A
28. C
29. B
30. D
31. B
32. B
33. D
34. D
35. A
36. C
37. C
38. A
39. B
40. C
41. D
42. D
43. B
44. D
45. B
46. B
47. D
48. C
49. A
50. C
51. C
52. C
53. A
54. A
55. D
56. D
57. B
58. A
59. B
60.
61. A
62. B
63. D
64. C
65. A
66. A
67. C
68. D
69. C
70. C
71. B
72. A
73. D
74. D
75. B
76. D
77. D
78. D
79. D
80. C
81. B
82. C
83. A
84. A
85. C
86. D
87. A
88. C
89. A
90. A
91. A
92. D
93. B
94. C
95. B
96. D
97. B
98. D
99. C
100. C

PREBOARD EXAM 3

Situation 1 - Concerted work efforts among members of the surgical team


is essential to the success of the surgical procedure.

1. The sterile nurse or sterile personnel touch only sterile


supplies and instruments. When there is a need for sterile supply
which is not in the sterile field, who hands out these items by
opening its outer cover?
a. Circulating nurse
b. Anesthesiologist
c. Surgeon
d. Nursing aide

2. The OR team performs distinct roles for one surgical procedure


to be accomplished within a prescribed time frame and deliver a
standard patient outcome. White the surgeon performs the surgical
procedure, who monitors the status of the client like urine
output, blood loss?

a. Scrub nurse
b. Surgeon
c. Anesthesiologist
d. Circulating nurse

3. Surgery schedules are communicated to the OR usually a day


prior to the procedure by the nurse of the floor or ward where the
patient is confined. For orthopedic cases, what department is
usually informed to be present in the OR?

a. Rehabilitation department
b. Laboratory department
c. Maintenance department
d. Radiology department

4. Minimally invasive surgery is very much into technology. Aside


from the usual surgical team who else to be present when a client
undergoes laparoscopic surgery?

a. Information technician
b. Biomedical technician
c. Electrician
d. Laboratory technicial

5. In massive blood loss, prompt replacement of compatible blood


is crucial. What department needs to be alerted to coordinate
closely with the patient's family for immediate blood component
therapy?

a. Security Division
b. Chaplaincy
c. Social Service Section
d. Pathology department

Situation 2 - You are assigned in the Orthopedic Ward where clients are
complaining of pain in varying degrees upon movement of body parts.

6. Troy is a one day post open reduction and internal fixation


(ORIF) of the left hip and is in pain. Which of the following
observation would prompt you to call the doctor?

a. Dressing is intact but partially soiled


b. Left foot is cold to touch and pedal pulse is absent
c. Left leg in limited functional anatomic position
d. BP 114/78, pulse of 82 beats/minute

7. There is an order of Demerol 50 mg I.M. now and every 6 hours p


r n. You injected Demerol at 5 pm. The next dose of Demerol 50 mg
I.M. is given:

a. When the client asks for the next dose


b. When the patient is in severe pain
c. At 11pm
d. At 12pm

8. You continuously evaluate the client's adaptation to pain.


Which of the following behaviors-indicate appropriate adaptation?

a. The client reports pain reduction and decreased activity


b. The client denies existence of pain
c. The client can distract himself during pain episodes
d. The client reports independence from watchers

9. Pain in Ortho cases may not be mainly due to the surgery. There
might be other factors such as cultural or psychological that
influence pain. How can you alter these factors as the nurse?

a. Explain all the possible interventions that may cause the client to
worry.
b. Establish trusting relationship by giving his medication on time
c. Stay with the client during pain episodes
d. Promote client's sense of control and participation in pain control
by listening to his concerns

10. In some hip surgeries, an epidural catheter for Fentanyl


epidural analgesia is given. What is your nursing priority care in
such a case?

a. Instruct client to observe strict bed rest


b. Check for epidural catheter drainage
c. Administer analgesia through epidural catheter as prescribed
d. Assess respiratory rate carefully
Situation 3 - Records are vital tools in any institution and should be
properly maintained for specific use and time.

11. The patient's medical record can work as a double-edged


swords. When can the medical record become the doctor's/nurse
worst enemy?

a. When the record is voluminous


b. When a medical record is subpoenaed in court
c. When it is missing
d. When the medical record is inaccurate, incomplete, and inadequate

12. Disposal of medical records in government


hospitals/institutions must be done in close coordination with
what agency?

a. Department of Interior and Local Government (DILG)


b. Metro Manila Development Authority (MMDA)
c. Records Management Archives Office (RMAO)
d. Depart of Health (DOH)

13. In the hospital, when you need-the medical record of a


discharged patient for research, you will request permission
through:

a. Doctor in charge
b. The hospital director
c. The nursing Service
d. Medical records section

14. You readmitted a client who was in another department a month


ago. Since you will need the previous chart, from whom do you
request the old chart?

a. Central supply section


b. Previous doctor's clinic
c. Department where the patient was previously admitted
d. Medical records section

15. Records Management and Archives Offices of the DOH is


responsible for implementing its policies on record, disposal. You
know that your institution is covered by this policy it;

a. Your hospital is considered tertiary


b. Your hospital is in Metro Manila
c. It obtained permit to operate from DOH
d. Your hospital is Philhealth accredited

Situation 4 - In the OR, there are safety protocols that should be


followed. The OR nurse should be well versed with all these to safeguard
the safety and quality to patient delivery outcome.

16. Which of the following should be given highest priority when


receiving patient in the OR?

a. Assess level of consciousness


b. Verify patient identification and informed consent
c. Assess vital signs
d. Check for jewelry, gown, manicure and dentures

17. Surgeries like I and D (incision and drainage) and debribement


are relatively short procedures but considered ‘dirty cases’.
When are these; procedures best scheduled?

a. Last case
b. In between cases
c. According to availability of anesthesiologist
d. According to the surgeon's preference

18. OR nurses should be aware that maintaining the client's safety


is the overall goal of nursing care during the intraoperative
phase. As the circulating nurse, you make certain that throughout
the procedure...

a. the surgeon greets his client before induction of anesthesia


b. the surgeon and anestheriologist are in tandem
c. strap made of strong non-abrasive material are fastened securely
around the joints of the knees and ankles and around the 2 hands around
an arm board
d. client is monitored throughout the surgery by the assistant
anesthesiologist

19. Another nursing check that should not be missed before the
induction of general anesthesia is:

a. check for presence underwear


b. check for presence dentures
c. check patient's
d. check baseline vital signs

20. Some different habits and hobbies affect postoperative


respiratory function. If your client smokes 3 packs of cigarettes
a day for the part 10 years, you will anticipate increased risk
for:

a. perioperative anxiety and stress


b. delayed coagulation time
c. delayed wound healing
d. postoperative respiratory function

Situation 5 - Nurses hold a variety of roles when providing care to a


perioperative patient.

21. Which of the following role would be the responsibility of the


scrub nurse?

a. Assess the readiness of the client prior to surgery


b. Ensure that the airway is adequate
c. Account for the number of sponges, needles, supplies, Used during the
surgical procedure
d. Evaluate the type of anesthesia appropriate for the surgical client

22. As a perioperative nurse, how can you best meet the safety
need of the client after administering preoperative narcotic?

a. Put side rails up and ask client not to get out of bed
b. Send the client to ORD with the family
c. Allow client to get up to go to the comfort room
d. Obtain consent form

23. It is the responsibility of the pre-op, nurse to do skin prep


for patients undergoing surgery. If hair at the operative site is
not shaved, what should be done to make suturing easy and lessen
chance of incision infection?

a. Draped
b. Pulled
c. Clipped
d. Shampooed

24. It is also the nurse's function to determine when infection is


developing in the surgical incision. The perioperative nurse
should observe for what signs of impending infection?

a. Localized heat and redness


b. Serosanguinous exudates and skin blanching
c. Separation of the incision
d. Blood clots and scar tissue are visible

25. Which of the following nursing intervention is done when


examining the incision wound and changing the dressing?

a. Observe the dressing and type and odor of drainage if any


b. Get patient's consent
c. Wash hands
d. Request the client to expose the incision wound

Situation 6 - Carlo, 16 years old, comes to the ER with acute asthmatic


attack. RR is 46/min and he appears to be in acute respiratory distress.

26. Which of She following nursing actions should be initiated


first?

a. Promote emotional support


b. Administer oxygen at 6L/min
c. Suction the client every 30 min
d. Administer bronchodilator by nebulizer

27. Aminophylline was ordered for acute asthmatic attack. The


mother asked the nurse, what its indication the nurse will say is:

a. Relax smooth muscles of the bronchial airway


b. Promote expectoration
c. Prevent thickening of secretions
d. Suppress cough

28. You will give health instructions to Carlo, a case of


bronchial asthma. The health instruction will include the
following EXCEPT:

a. Avoid emotional stress and extreme temperature


b. Avoid pollution like smoking
c. Avoid pollens, dust seafood
d. Practice respiratory isolation

29. The asthmatic client asked you what breathing technique he can
best practice when asthmatic attack starts. What will be the best
position?

a. Sit in high-Fowler's position with extended legs


b. Sit-up with shoulders back
c. Push on abdomen during exhalation
d. Lean forward 30-40 degrees with each exhalation

30. As a nurse you are always alerted to monitor status


asthmaticus who will likely and initially manifest symptoms of:
a. metabolic alkalosis
b. respiratory acidosis
c. respiratory alkalosis
d. metabolic acidosis

Situation 7 - Joint Commission on Accreditation of Hospital Organization


(JCAHP) patient safety goals and requirements include the care and
efficient use of technology in the OR arid elsewhere in the healthcare
facility.

31. As the head nurse in the OR, how can you improve the
effectiveness of clinical alarm systems?

a. limit suppliers to a few so that quality is maintained


b. implement a regular inventory of supplies and equipment
c. Adherence to manufacturer's recommendation
d. Implement a regular maintenance and testing of alarm systems

32. Over dosage of medication or anesthetic can happen even with


the aid of technology like infusion pump, sphymomanometer, and
similar devices/machines. As a staff, how can you improve the
safety of using infusion pumps?

a. Check the functionality of the pump before use


b. Select your brand of infusion pump like you do with your cellphone
C. Allow the technician to set the; infusion pump before use
d. Verify the flow rate against your computation

33. JCAHOs universal protocol for surgical and invasive procedures


to prevent wrong site, wrong person, and wrong procedures/surgery
includes the following EXCEPT:

a. Mark the operative site if possible


b. Conduct pre-procedure verification process
c. Take a video of the entire intra-operative procedure
d. Conduct time out immediately before starting the procedure

34. You identified a potential risk of pre and post operative


clients. To reduce the risk of patient harm resulting from fall,
you can implement the following EXCEPT:

a. Assess potential risk of fail associated with the patient's the


following EXCEPT: medication regimen
b. Take action to address any identified risks through Incident Report
(IR)
c. Allow client to walk with relative to the OF?
d. Assess and periodically reassess individual client's risk for falling
35. As a nurse you know you can improve on accuracy of patient's
identification by 2 patient identifiers, EXCEPT:

a. identify the client by his/her wrist tag and verify with family
members
b. identify client by his/her wrist tag and call his/her by name
c. call the client by his/her case and bed number
d. call the patient by his/her name and bed number

Situation 8 - Team efforts is best demonstrated in the OR

36. If you are the nurse in charge for scheduling surgical cases,
what important information do you need to ask the surgeon?

a. Who is your internist


b. Who is your assistant and anesthesiologist, and what is your
preferred time and type of surgery?
c. Who are your anesthesiologist, internist, and assistant
d. Who is your anesthesiologist.

37. In the OR, the nursing tandem for every surgery is:

a. Instrument technician and circulating nurse


b. Nurse anesthetist, nurse assistant, and instrument technician
c. Scrub nurse and nurse anesthetist
d. Scrub and circulating nurses

38. While team effort is needed in the OR for efficient and


quality patient care delivery, we should limit the number of
people in the room for infection control. Who comprise this team?

a. Surgeon, anesthesiologist, scrub nurse, radiologist, orderly


b. Surgeon, assistants, scrub nurse, circulating nurse, anesthesiologist

c. Surgeon, assistant surgeon, anesthesiologist, scrub nurse,


pathologist
d. Surgeon, assistant surgeon, anesthesiologist, intern, scrub nurse

39. When surgery is on-going, who coordinates the activities


outside, including the family?

a. Orderly/clerk
b. Nurse supervisor
c. Circulating nurse
d. Anaesthesiologist
40. The breakdown in teamwork is often times a failure in:

a. Electricity
b. Inadequate supply
c. Leg work
d. Communication

Situation 9 - Colostomy is a surgically created anus- It can be


temporary or permanent, depending on the disease condition.

41. Skin care around the stoma is critical. Which of the following
is not indicated as a skin care barriers?
a. Apply liberal amount of mineral oil to the area
b. Use karaya paste and rings around the stoma
c. Clean the area daily with soap and water before applying bag
d. Apply talcum powder twice a day

42. What health instruction will enhance regulation of a colostomy


(defecation) of clients?

a. Irrigate after lunch everyday


b. Eat fruits and vegetables in all three meals
c. Eat balanced meals at regular intervals
d. Restrict exercise to walking only

43. After ileostomy, which of the following condition is NOT


expected?

a. increased weight
b. Irritation of skin around the stoma
c. Liquid stool
d. Establishment of regular bowel movement

44. The following are appropriate nursing interventions during


colostomy irrigation EXCEPT:

a. Increase the irrigating solution flow rate when abdominal cramps is


felt
b. Insert 2-4 inches of an adequately lubricated catheter to the stoma
c. Position client in semi-Fowler
d. Hand the solution 18 inches above the stoma

45. What sensation is used as a gauge so that patients with


ileostomy can determine how often their pouch should be drained?

a. Sensation of taste
b. Sensation of pressure
c. Sensation of smell
d. Urge to defecate

Situation 10 - As a beginner in research, you are aware that sampling is


an essential element of the research process.

46. What does a sample group represent?

a. Control group
b. Study subjects
c. General population
d. Universe

47. What is the most important characteristics of a sample?

a. Randomization
b. Appropriate location
c. Appropriate number
d. Representativeness

48. Random sampling ensures that each subject has:

a. Been selected systematically


b. An equal change of selection
c. Been selected based on set criteria
d. Characteristics that match other samples

49. Which of the following sampling methods allows the use of any
group of research subject?

a. Purposive
b. Convenience
c. Snow-bail
d. Quota

50. You decided to include 5 barangays in your municipality and


chose a sampling method that would get representative samples from
each barangay. What should be the appropriate method for you to
use in this care?

a. Cluster sampling
b. Random sampling
c. Stratifies sampling
d. Systematic sampling

Situation 11 -After an abdominal surgery, the circulating and scrub


nurses have critical responsibility about sponge and Instrument count.
51. When is the first sponge/instrument count reported?

a. Before closing the subcutaneous layer


b. Before peritoneum is closed
c. Before dosing the skin
d. Before the fascia is sutured

52. What major supportive layer of the abdominal wall must be


sutured with long tensile strength such as cotton or nylon or silk
suture?

a. Fascia
b. Muscle
c. Peritoneum
d. Skin

53. Like sutures, needles also vary in shape and uses. If you are
the scrub nurse for a patient who is prone to keloid formation and
has a low threshold of pain, what needle would you prepare?

a. Round needle
b. A traumatic needle
c. Reverse cutting needle
d. Tapered needle

54. Another alternative "suture" for skin closure is the use of


_______________:

a. Staple
b. Therapeutic glue
c. Absorbent dressing
d. invisible suture

55. Like any nursing interventions, counts should be documented.


To whom does the scrub nurse report any discrepancy of country so
that immediate 'and appropriate action in instituted?

a. Anesthesiologist
b. Surgeon
c. Or nurse supervisor
d. Circulating nurse

Situation 12 - As a nurse, you should be aware and prepared of the


different roles you play.

56. What role do you play, when you hold all clients’ information
entrusted to you in the strictest confidence?

a. Patient's advocate
b. Educator
c. Patient's Liaison
d. Patient's arbiter

57. As a nurse, you can help improve the effectiveness of


communication among healthcare givers

a. Use of reminders of what to do


b. Using standardized list of abbreviations, acronyms, and symbols
c. One-on-one oral endorsement
d. Text messaging and e-mail

58. As a nurse, your primary focus in the workplace is the


client's safety. However, personal safety is also a concern. You
can communicate hazards to your co-workers through the use of the
following EXCEPT:

a. Formal training
b. Posters
c. Posting IR in the bulletin board
d. Use of labels and signs

59. As a nurse, what is one of the best way to reconcile


medications across the continuum of care?

a. Endorse on a case-to-case basis


b. Communication a complete list of the patient's medication to the next
provider of service
c. Endorse in writing
d. Endorse the routine and 'stat' medications every shift

60. As a nurse, you protect yourself and co-workers from


misinformation and misrepresentations through the following
EXCEPT:

a. Provide information to clients about a variety of services that can


help alleviate the client's pain and other conditions
b. Advising the client, by virtue of your expertise, that which can
contribute to the client's well-being
c. Health education among clients and significant others regarding the
use of chemical disinfectant
d. Endorsement thru trimedia to advertise your favorite disinfectant
solution
61. A one-day postoperative abdominal surgery client has been
complaining of severe throbbing abdominal pain described as 9 in a
1-10 pain rating. Your assessment reveals bowel sounds on all
quadrants and the dressing is dry and intact. What nursing
intervention would you take?

a. Medicate client as prescribed


b. Encourage client to do imagery
c. Encourage deep breathing and turning
d. Call surgeon stat

62. Pentoxicodone 5 mg IV every 8 hours was prescribed for post


abdominal pain. Which will be your priority nursing action?

a. Check abdominal dressing for possible swelling


b. Explain the proper use of PCA to alleviate anxiety
c. Avoid overdosing to prevent dependence/tolerance
d. Monitor VS, more importantly RR .

63. The client complained of abdominal and pain. Your nursing


intervention that can alleviate pain is:

a. Instruct client to go to sleep and relax


b. Advice the client to close the lips and avoid deep breathing and
talking
c. Offer hot and clear soup
d. Turn to sides frequently and avoid too much talking

64. Surgical pain might be minimized by which nursing action in


the OR:

a. Skill of surgical team and lesser manipulation


b. Appropriate preparation For the scheduled procedure
c. Use of modem technology in closing the wound
d. Proper positioning and draping of clients

65. One very common cause of postoperative pain is:

a. Forceful traction during surgery


b. Prolonged surgery
c. Break in aseptic technique
d. Inadequate anesthetic

Situation 14 - You were on duty at the medical ward when Zeny came in
for admission for tiredness, cold intolerance, constipation, and weight
gain. Upon examination, the doctor's diagnosis was hypothyroidism.
66. Your independent nursing care for hypothyroidism includes:

a. administer sedative round the clock


b. administer thyroid hormone replacement
c. providing a cool, quiet, and comfortable environment
d. encourage to drink 6-8 glasses of water

67. As the nurse, you should anticipate to administer which of the


following medications to Zeny who is diagnosed to be suffering
from hypothyroidism?

a. Levothyroxine
b. Lidocaine
c. Lipitor
d. Levophed

68. Your appropriate nursing diagnosis for Zeny who is suffering


from hypothyroidism would probably include which of the following?

a. Activity intolerance related to tiredness associated with disorder


b. Risk to injury related to incomplete eyelid closure
c. Imbalance nutrition related to hypermetabolism
d. Deficient fluid volume related to diarrhea

69. Myxedema coma is a life threatening complication of long


standing and untreated hypothyroidism with one of the following
characteristics.

a. Hyperglycemia
b. hypothermia
c. hyperthermia
d. hypoglycemia

70. As a nurse, you know that the most common type of goiter is
related to a deficiency

a. thyroxine
b. thyrotropin
c. iron
d. iodine

Situation 15 - Mrs. Pichay is admitted to your ward. The MD ordered


"Prepared for thoracentesis this pm to remove excess air from the
pleural cavity."

71. Which of the following nursing responsibility is essential in


Mrs. Pichay who will undergo thoracentesis?
a. Support, and reassure client during the procedure
b. Ensure that informed consent has been signed
c. Determine if client has allergic reaction to local anesthesia
d. Ascertain if chest x-rays and other tests have been prescribed and
completed

72. Mrs. Pichay who is for thoracentesis is assisted by the nurse


to any of the following positions, EXCEPT:

a. straddling a chair with arms and head resting on the back of the
chair
b. lying on the unaffected side with the bed elevated 30-40 degrees
c. lying prone with the head of the bed lowered 15-30 degrees
d. sitting on the edge of the bed with her feet supported and arms and
head on a padded overhead table

73. During thoracentesis, which of the following nursing


intervention will be most crucial?

a. Place patient in a quiet and cool room


b. Maintain strict aseptic technique
c. Advice patient to sit perfectly still during needle insertion until
it has been withdrawn from the chest
d. Apply pressure over the puncture site as soon as the needle is
withdrawn

74. To prevent leakage of fluid in the thoracic cavity, how wilt


you position the client after thoracentesis?

a. Place flat in bed


b. Turn on the unaffected side
c. Turn on the affected side
d. On bed rest

75. Chest x-ray was ordered after thoracentesis. When you client
asks what is the reason for another chest x-ray, you will explain:

a. to rule out pneumothorax


b. to rule out any possible perforation
c. to decongest
d. to rule out any foreign: body

Situation 16 - In the hospital, you are aware that we are helped by


the .use of a variety of equipment/devices to enhance quality patient
care delivery;
76. You are initiate an IV line to your patient, Kyle, 5, who is
febrile. What IV administration set will you prepare?

a. Blood transfusion set


b. Macroset
c. Volumetric chamber
d. Microset

77. Kyle is diagnosed to have measles. What will your protective


personal attire include?

a. Gown
b. Eyewear
c. Face mask
d. Gloves

78. What will you do to ensure that Kyle, who is febrile, will
have a liberal oral fluid intake?

a. Provide a glass of fruit every meal


b. Regulate his IV to 30 drops per minute
c. Provide a calibrated pitcher of drinking water and juice at the
bedside and monitor intake and output
d. Provide a writing pad to record his intake

79. Before bedtime, you went to ensure Kyle's safety in 'bed. You
will do which of the following:

a. Put the lights on


b. Put the side rails up
c. Test the call system
d. Lock the doors

80. Kyle's room is fully mechanized. What do you teach the watcher
and Kyle to alert the nurse for help?

a. How to lock side rails


b. Number of the telephone operator
c. Call system
d. Remote control

Situation 17 - Tony, 11 years old, has 'kissing tonsils' and is


scheduled for tonsillectomy and adenoidectomy or T and A.

81. You are the nurse of Tony who will undergo T and A in the
morning. His mother asked you if Tony will be put to sleep. Your
teaching will focus on:
a. spinal anesthesia
b. anesthesiologist’s preference
c. local anesthesia
d. general anesthesia

82. Mothers of children undergoing tonsillectomy and adenoidectomy


usually ask what food prepared and give their children after
surgery. You as the nurse will say:

a. balanced diet when fully awake


b. hot soup when awake
c. ice cream when fully awake
d. soft diet when fully awake

83. The RR nurse should monitor for the most common postoperative
complication of:

a. hemorrhage
b. endotracheal tube perforation
c. esopharyngeal edema
d. epiglottis

84. The PACU nurse will maintain postoperative T and A client in


what position?

a. Supine with neck hyperextended and supported with pillow


b. Prone with the head on pillow and tuned to the side
c. Semi-Fowler's with neck flexed
d. Reverse trendelenburg with extended neck

85. Tony is to be discharged in the afternoon of the same day


after tonsillectomy and adenoidectomy. You as the RN will make
sure that the family knows to:

a. offer osteorized feeding


b. offer soft foods for a week to minimize discomfort while swallowing
c. supplement his diet with vitamin C rich juices to enhance heating
d. offer clear liquid for 3 days to prevent irritation

Situation 18 - Rudy was diagnosed to have chronic renal failure.


Hemodialysis is ordered that an A-V shunt was surgically created.

86. Which of the following action would be of highest priority


with regards to the external shunt?

a. Avoid taking BP or blood sample from the arm with shunt


b. Instruct the client not to exercise the arm with the shunt
c. Heparinize the shunt daily
d. Change dressing of the shunt daily

87. Diet therapy for Rudy, who has acute renal failure, is tow-
protein, low potassium and sodium. The nutrition instruction
should include:

a. Recommend protein of high biologic value like eggs, poultry and lean
meat
b. Encourage client to include raw cucumbers, carrot, cabbage, and
tomatoes
c. Allowing the client cheese, canned foods, and other processed food
d. Bananas, cantaloupe, orange and other fresh fruits can be included in
the diet

88. Rudy undergoes hemodialysis for the first time and was scared
of disequilibrium syndrome. He asked you how this can be
prevented. Your response is:

a. maintain a conducive comfortable and cool environment


b. maintain fluid and electrolyte balance
c. initial hemodialysis shall be done for 30 minutes only so as not to
rapidly remove the waste from the blood than from the brain
d. maintain aseptic technique throughout the hemodialysis

89. You are assisted by a nursing aide with the care of the client
with renal failure. Which delegated function to the aide would you
particularly check?

a. Monitoring and recording I and O


b. Checking bowel movement
c. Obtaining vital signs
d. Monitoring diet

90. A renal failure patient was ordered for creatinine clearance.


As the nurse you will collect

a. 48 jour urine specimen


b. first morning urine
c. 24 hour urine specimen
d. random urine specimen

Situation 19 - Fe is experiencing left sharp pain and occasional


hematuria. She was advised to undergo IVP by her physician.

91. Fe was so anxious about the procedure and particularly


expressed her low pain threshold. Nursing health instruction will
include:

a. assure the client that the pain is associated with the warm sensation
during the administration of the Hypaque by IV
b. assure the client that the procedure painless
c. assure the client that contrast medium will be given orally
d. assure the client that x-ray procedure like IVP is only done by
experts

92. What will the nurse monitor and instruct the client and
significant others, post IVP?

a. Report signs and symptoms for delayed allergic reactions


b. Observe NPO for 6 hours
c. Increase fluid intake
d. Monitor intake and output

93. Post IVP, Fe should excrete the contrast medium. You


instructed the family to include more vegetables in the diet and

a. increase fluid intake


b. barium enema
c. cleansing enema
d. gastric lavage

94. The IVP reveals that Fe has small renal calculus that can be
passed out spontaneously. To increase the chance of passing the
stones, you instructed her to force fluids and do which of the
following?

a. Balanced diet
b. Ambulance more
c. Strain all urine
d. Bed rest

95. The presence of calculi in the urinary tract is called:

a. Colelithiasis
b. Nephrolithiasis
c. Ureterolithiasis
d. Urolithiasis

Situation 20 - At the medical-surgical ward, the nurse must also be


concerned about drug interactions.

96. You have a client with TPN. You know that in TPN, like blood
transfusion, there should be no drug incorporation. However, the
MD's order read; incorporate insulin to present TPN. Will you
follow the order?

a. No, because insulin will induce hyperglycemia in patients with TPN


b. Yes, because insulin is chemically stable with TPN and can enhance
blood glucose level
c. No, because insulin is not compatible with TPN
d. Yes, because it was ordered by the MD

97. The RN should also know that some drugs have increased
absorption when infused in PVC container. How will you administer
drugs such as insulin, nitroglycerine hydralazine to promote
better therapeutic drug effects?

a. Administer by fast drip


b. Inject the drugs as close to the IV injection site
c. Incorporate to the IV solution
d. Use volumetric chamber

98. One patient has a 'runaway' IV of 50% dextrose. To prevent


temporary excess of insulin transient hyperinsulin reaction, what
solution should you prepare in anticipation of the doctors order?

a. Any IV solution available to KVO


b. Isotonic solution
c. Hypertonic solution
d. Hypotonic solution

99. How can nurse prevent drug interaction including absorption?

a. Always flush with NSS after IV administration


b. Administering drugs with more diluents
c. Improving on preparation techniques
d. Referring to manufacturer's guidelines

100. In insulin administration, it should be understood that our


body normally releases insulin according to our blood glucose
level. When is insulin and glucose level highest?

a. After excitement
b. After a good night's rest
c. After an exercise
d. After ingestion of food

PREBOARD EXAM 4
CARE OF CLIENTS WITH PHYSIOLOGIC AND PSYCHOSOCIAL ALTERATIONS

Situation 1 - Because of the serious consequences of severe burns


management requires a multi disciplinary approach. You have important
responsibilities as a nurse.

1. While Sergio was lighting a barbecue grill with a lighter


fluid, his shirt burst into flames. The most effective way to
extinguish the flames with as little further damage as possible is
to:

a. log roll on the grass/ground


b. slap the flames with his hands
c. remove the burning clothes
d. pour cold liquid over the flames

2. Once the flames are extinguished, it is most important to:

a. cover Sergio with a warm blanket


b. give him sips of water
c. calculate the extent of his burns
d. assess the Sergio's breathing

3. Sergio is brought to the Emergency Room after the barbecue


grill accident. Based on the assessment of the physician, Sergio
sustained superficial partial thickness bums on his trunk, right
upper extremities ad right lower extremities. His wife asks what
that means. Your most accurate response would be:

a. Structures beneath the skin are damaged


b. Dermis is partially damaged
c. Epidermis and dermis are both damaged
d. Epidermis is damaged

4. During the first 24 hours after thermal injury, you should


assess Sergio for

a. hypokalemia and hypernatremia


b. hypokalemia and hyponatremia
c. hyperkalemia and hyponatremia
d. hyperkalemia and hypernatremia

5. Teddy, who sustained deep partial thickness and full thickness


burns of the face, whole anterior chest and both upper extremities
two days ago, begins to exhibit extreme restlessness. You
recognize that this most likely indicates that Teddy is
developing:
a. Cerebral hypoxia
b. Hypervolemia
c. Metabolic acidosis
d. Renal failure .

Situation 2 - You are now working as a staff nurse in a general


hospital. You have to be prepared to handle situations with ethico-legal
and moral implications.

6. You are on night duty in the surgical ward. One of our patients
Martin is prisoner who sustained an abdominal gunshot wound. He is
being guarded by policemen from the local police unit. During your
rounds you heard a commotion. You saw the policeman trying to hit
Martin. You asked why he was trying to hurt Martin. He denied the
matter. Which among the following activities will you do first?

a. Write an incident report


b. Call security officer and report the incident
c. Call your nurse supervisor and report the incident :
d. Call the physician on duty

7. You are on morning duty in the medical ward. You have 10


patients assigned to you. During your endorsement rounds, you
found out that one of your patients was not in bed. The patient
next to him informed you that he went home without notifying the
nurses. Which among the following will you do first?

a. Make and incident report


b. Call security to report the incident
c. Wait for 2 hours before reporting
d. Report the incident to your supervisor

8. You are on duty in the medical ward. You were asked to check
the narcotics cabinet. You found out that what is on record does
not tally with the drugs used. Which among the following will you
do first?

a. Write an incident report and refer the matter to the nursing director
b. Keep your findings to yourself
c. Report the matter to your supervisor
d. Find out from the endorsement any patient who might have been given
narcotics

9. You are on duty in the medical ward. The mother of your patient
who is also a nurse came running to the nurse station and informed
you that Fiolo went into cardiopulmonary arrest. Which among the
following will you do first?

a. Start basic life support measures


b. Call for the Code
c. Bring the crush cart to the room
d. Go to see Fiolo and assess for airway patency and breathing problems

10. You are admitting Jorge to the ward and you found out that he
is positive for HIV. Which among the following will you do first?

a. Take note of it and plan to endorse this to next shift


b. Keep this matter to your self
c. Write an incident report
d. Report the matter to your head nurse

Situation 3 - Colorectal cancer can affect old and younger people.


Surgical procedures and other modes of treatment are done to ensure
quality of life. You are assigned in the Cancer institute to care of
patients with this type of cancer.

11. Larry, 55 years old, who is suspected of having colorectal


cancer, is admitted to the CI. After taking the history and vital
signs the physician does which test as a screening test for
colorectal cancer.

a. Barium enema
b. Carcinoembryonig antigen
c. Annual digital rectal examination
d. Proctosigmoidoscopy

12. To confirm his impression of colorectal cancer, Larry will


require which diagnostic study?

a. carcinoembryonic antigen
b. proctosigmoidbscopy
c. stool hematologic test
d. abdominal computed tomography (CT) test

13. The following are risk factors for colorectal cancer, EXCEPT:

a. inflammatory bowels
b. high fat, high fiver diet
c. smoking
d. genetic factors-familial adenomatous polyposis

14. Symptoms associated with cancer of the colon include:


a. constipation, ascites and mucus in the stool
b. diarrhea, heartburn and eructation
c. blood in the stools, anemia, and pencil-shaped, stools
d. anorexia, hematemesis, and increased peristalsis

15. Several days prior to bowel surgery, Larry may be given


sulfasuxidine and neomycin primarily to:

a. promote rest of the bowel by minimizing peristalsis


b. reduce the bacterial content of the colon
c. empty the bowel of solid waste
d. soften the stool by retaining water in the colon

Situation 4 - ENTEROSTOMAL THERAPY is now considered especially in


nursing. You are participating in the OSTOMY CARE CLASS.

16. You plan to teach Fermin how to irrigate the colostomy when:

a. The perineal wound heals and Fermin can sit comfortably on the
commode
b. Fermin can lie on the side comfortably, about the 3rd postoperative
day
c. The abdominal incision is close and contamination is no longer a
danger
d. The stool starts to become formed, around the 7th postoperative day

17. When preparing to teach Fermin how to irrigate his colostomy,


you should plan to do the procedure:

a. When Fermin would have normal bowel movement


b. At least 2 hours before visiting hours
c. Prior to breakfast and morning care
d. After Fermin accepts alteration in body image

18. When observing a rectum demonstration of colostomy irrigation,


you know that more teaching is required if Fermin:

a. Lubricates the tip of the catheter prior to inserting into the stoma
b. Hands the irrigating bag on the bathroom door doth hook during fluid
insertion
c. Discontinues the insertion of fluid after only 500 ml of fluid had
been insertion
d. Clamps off the flow of fluid when feeling uncomfortable

19. You are aware that teaching about colostomy care is understood
when Fermin states, "I will contact my physician and report:
a. If I have any difficulty inserting the irrigating tub into the
stoma.”
b. If I notice a loss of sensation to touch in the stoma tissue."
c. The expulsion of flatus while the irrigating fluid is running out."
d. When mucus is passed from the stoma between irrigation."

20. You would know after teaching. Fermin that dietary instruction
for him is effective when he states, "It is important that I eat:

a. Soft foods that are easily digested and absorbed by my large


intestine."
b. Bland food so that my intestines do not become irritate."
c. Food low in fiber so that there is less stool."
d. Everything that I ate before the operation, while avoiding foods that
cause gas."

Situation 5 - Ensuring safety is one of your most important


responsibilities. You will need to provide instructions and information
to your clients to prevent complications.

21. Randy has chest tubes attached to a pleural drainage system.


When caring for him you should:

a. empty the drainage system at the end of the shift


b. clamp the chest tube when auctioning
c. palpate the surrounding areas for crepitus
d. change the dressing daily using aseptic techniques

22. Fanny came in from PACU after pelvic surgery. As Fanny's nurse
you know that the sign that would be indicative of a developing
thrombophlebitis would be:

a. a tender, painful area on the leg


b. a pitting edema of the ankle
c. a reddened area at the ankle
d. pruritus on the calf and ankle

23. To prevent recurrent attacks on Terry who has acute


glumerulonephritis, you should instruct her to:

a. seek early treatment for respiratory infections


b. take showers instead of tub bath
c. continue to take the same restrictions on fluid intake
d. avoid situations that involve physical activity

24. Herbert has a laryngectomy and he is now for discharge. Re


verbalized his concern regarding his laryngectomy tube being
dislodged. What should you teach him first?

a. Recognize that prompt closure of the tracheal opening may occur


b. Keep calm because there is no immediate emergency
c. Reinsert another tubing immediately
d. Notify the physician at once

25. When caring for Larry after an exploratory chest surgery and
pneumonectomy, your priority would be to maintain:

a. supplementary oxygen
b. ventilation exchange
c. chest tube drainage
d. blood replacement

Situation 6 - Infection can cause debilitating consequences when host


resistance is compromised and virulence of microorganisms and
environmental factors are favorable. Infection control is one important
responsibility of the nurse to ensure quality of care.

26. Honrad, who has been complaining of anorexia and feeling


tired, develops jaundice. After a workup he is diagnosed of having
Hepatitis A. His wife asks you about gamma globulin for herself
and her household help. Your most appropriate response would be:

a. "Don't worry your husband's type of hepatitis is no longer


communicable"
b. "Gamma globulin provides passive immunity for Hepatitis B"
c. "You should contact your physician immediately about getting gamma
globulin."
d. "A vaccine has been developed for this type of hepatitis"

27. Voltaire develops a nosocomial respiratory tract infection. He


asks you what that means.

a. "You acquired the infection after you have been admitted to the
hospital."
b. "This is a highly contagious infection requiring complete isolation."
c. "The infection you had prior to hospitalization flared up."
d. "As a result of medical treatment, you have acquired a secondary
infection.''

28. As a nurse you know that one of the complications that you
have to watch out for when caring for Omar who is receiving total
parenteral nutrition is:

a. stomatitis
b. hepatitis
c. dysrhythmia
d. infection

29. A solution used to treat Pseudomonas would infection is:

a. Dakin's solution
b. Half-strength hydrogen peroxide
b. Acetic acid
d. Betadine

30. Which of the following is most reliable in diagnosing a wound


infection?

a. Culture and sensitivity


b. Purulent drainage from a wound
c. WBC count of 20,000/pL
d. Gram stain testing

Situation 7 - As a nurse you need to anticipate the occurrence of


complications of stroke so that life threatening situations can be
prevented.

31. Wendy is admitted to the hospital with signs and symptoms of


stroke. Her Glasgow Coma Scale is 6 on admission. A central venous
catheter was inserted and an I.V. infusion was started. As a nurse
assigned to Wendy what will he your priority goal?

a. Prevent skin breakdown


b. Preserve muscle function
c. Promote urinary elimination
d. Maintain a patent airway

32. Knowing that for a comatose patient hearing is the best last
sense to be lost, as Judy's nurse, what should you do?

a. Tell her family that probably she can't hear them


b. Talk loudly so that Wendy can hear you
c. Tell her family who are in the room not to talk
d. Speak softly then hold her hands gently

33. Which among the following interventions should you consider as


the highest priority when caring for June who has hemiparersis
secondary to stroke?

a. Place June on an upright lateral position


b. Perform range of motion exercises
c. Apply antiembolic stocking
d. Use hand rolls or pillows for support

34. Ivy, age 40, was admitted to the hospital with a severe
headache, stiff neck and photophobia. She was diagnosed with a
subarachnoid hemorrhage secondary to ruptured aneurysm. While
waiting for surgery, you can provide a therapeutic by doing which
of the following?

a. honoring her request for a television


b. placing her bed near the window
c. dimming the light in her room
d. allowing the family unrestricted visiting privileges

35. When performing a neurological assessment on Walter, you find


that his pupils are fixed and dilated. This indicated that he:

a. probably has meningitis


b. is going to be blind because of trauma
c. is permanently paralyzed
d. has received a significant brain injury

Situation 8 - With the improvement in life expectancies and the emphasis


in the quality of life it is important to provide quality care to our
older patients. There are frequently encountered situations and issues
relevant to the older, patients.

36. Hypoxia may occur in the older patients because of which of


the following physiologic changer associated with aging.

a Ineffective airway clearance


b. Decreased alveolar surface area
c. Decreased anterior-posterior chest diameter
d. Hyperventilation

37. The older patient is at higher risk for in inconvenience


because of:

a. dilated urethra
b. increased glomerular filtration rate
c. diuretic use
d. decreased bladder capacity

38. Merle, age 86, is complaining of dizziness when she stands up.
This may indicate:

a. dementia
b. a visual problem
c. functional decline
d. drug toxicity

39. Cardiac ischemia in an older patient usually produces:

a. ST-T wave changes


b. Very high creatinine kinase level
c. chest pain radiating to the left arm
d. acute confusion

40. The most dependable sign of infection in the older patient is:

a. change in mental status pain


b. fever
c. pain
d. decreased breath sound with crackles

Situation 9 - A "disaster" is a large-scale emergency—even a small


emergency left unmanaged may turn into a disaster. Disaster preparedness
is crucial and is everybody's business. There are agencies that are in
charge of ensuring prompt response. Comprehensive Emergency Management
(CEM) is an integrated approach to the management of emergency program
and activities for all four emergency phases (mitigation, preparedness,
response, and recovery), for all type of emergencies and disasters
(natural, man-made, and attack) and for all levels of government and the
private sector.

41. Which of the four phases of emergency management is defined as


"sustained action that reduces or eliminates long-term risk to
people and properly from natural hazards and the effect"?

a. Recovery
b. Mitigation
c. Response
d. Preparedness

42. You are a community health nurse collaborating with the Red
Cross and working with disaster relief following a typhoon which
flooded and devastated the whole province. Finding safe housing
for survivors, organizing support for the family, organizing
counseling debriefing sessions and securing physical care are the
services you are involved with. To which type of prevention are
these activities included.

a. Tertiary prevention
b. Primary prevention
c. Aggregate care prevention
d. Secondary prevention

43. During the disaster you see a victim with a green tag, you
know that the person:

a. has injuries that are significant and require medical care but can
wait hours will threat to life or limb
b. has injuries that are life threatening but survival is good with
minimal intervention
c. indicates injuries that are extensive and chances of survival are
unlikely even with definitive care
d. has injuries that are minor and treatment can be delayed from hours
to days

44. The term given to a category of triage that refers to life


threatening or potentially life threatening injury or illness
requiring immediate treatment:

a. Immediate
b. Emergent
c. Non-acute
d. Urgent

45. Which of the following terms refer to a process by which the


individual receives education about recognition of stress
reactions and management strategies for handling stress which may
be instituted after a disaster?

a. Critical incident stress management


b. Follow-up
c. Defriefing
d. Defusion

Situation 10 - As a member of the health and nursing team you have a


crucial role to play in ensuring that all the members participate
actively is the various tasks agreed upon,

46. While eating his meal, Matthew accidentally dislodges his IV


line and bleeds. Blood oozes on the surface of the over-bed table.
It is most appropriate that you instruct the housekeeper to clean
the table with:

a. Acetone
b. Alcohol
c. Ammonia
d. Bleach

47. You are a member of the infection control team, of the


hospital. Based on a feedback during the meeting of the committee
there is an increased incidence of pseudomonas infection in the
Burn Unit (3 out of 10 patients had positive blood and wound
culture). What is your priority activity?

a. Establish policies for surveillance and monitoring


b. Do data gathering about the possible sources of infection
(observation, chart review, interview)
c. Assign point persons who can implement policies
d. Meet with the nursing group working in the burn unit and discuss
problem with them feel

48. Part of your responsibility as a member of the diabetes core


group is to get referrals from the various wards regarding
diabetic patients needing diabetes education. Prior to discharge
today 4 patients are referred to you. How would you start
prioritizing your activities?

a. Bring your diabetes teaching kit and start your session taking into
consideration their distance from your office
b. Contact the nurse-in-charge and find out from her the reason for the
referral
c. Determine their learning needs then prioritize
d. involve the whole family in the teaching class

49. You have been designated as a member of the task force to plan
activities for the Cancer Consciousness Week. Your committee has 4
months to plan and implement the plan. You are assigned to contact
the various cancer support groups in your hospital. What will be
your priority activity?

a. Find out if there is a budget for this activity


b. Clarify objectives of the activity with the task force before
contacting the support groups
c. Determine the VIPs and Celebrities who will be invited
d. Find out how many support groups there are in the hospital and get
the contact number of their president

50. You are invited to participate in the medical mission activity


of your alumni association. In the planning stage everybody is
expected to identify what they can do during the medical mission
and what resources are needed. You though it is also your chance
to share what you can do for others. What will be your most
important role where you can demonstrate the impact of nursing
health?

a. Conduct health education on healthy lifestyle


b. Be a triage nurse
c. Take the initial history and document findings
d. Act as a coordinator

Situation 11 - One of the realities that we are confronted with is'6w


mortality. It is important for us nurses to be aware of how we view
suffering, pain, illness, and even our death as well as its meaning.
That way we can help our patients cope with death and dying.

51. Irma is terminally ill she speaks to you in confidence. You


now feel that Irma's family could be helpful if they knew what
Irma has told you. What should you do first?

a. Tell the physician who in turn could tell the family


b. Obtain Irma's permission to share the information with the family
c. Tell Irma that she has to tell her family what she told you
d. Make an appointment to discuss the situation with the family

52. Ruby who has been told she has terminal cancer turns away aha
refuses to respond to you. You can best help her by:

a. Coming back periodically and indicating your availability if she


would like you to sit with her
b. Insisting that Ruby should talk with you because it is not good to
Keep everything inside
c. Leaving her atone because she is uncooperative and unpleasant to be
with
d. Encouraging her to be physically active as possible

53. Leo who is terminally ill and recognizes that he is in the


process of losing, everything and everybody he loves, is
depressed. Which of the following would best help him during
depression?

a. Arrange for visitors who might cheer him


b. Sit down and talk with him for a while
c. Encourage him to look at the brighter side of things
d. Sit silently with him

54. Which of the following statements would best indicate that


Ruffy; who is dying has accepted this impending death?

a. "I'm ready to do."


b. "I have resigned myself to dying"
c. "What's the use"?
d: "I'm giving up"

55. Maria, 90 years old has planned ahead for her-death-


philosophically, socially, financially and emotionally. This is
recognized as:

a. Acceptance that death is inevitable


b Avoidance of the true sedation
c. Denial with planning for continued life
d. Awareness that death will soon occur

Situation 12 - Brain tumor, whether malignant or benign, has serious


management implications nurse, you should be able to understand the
consequences of the disease and the treatment.

56. You are caring for Conrad who has a brain tumor and increased
intracranial Pressure (ICP). Which intervention should you include
in your plan to reduce ICP?

a. Administer bowel! Softener


b. Position Conrad with his head turned toward the side of the tumor
c. Provide sensory stimulation
d. Encourage coughing and deep breathing

57. Keeping Conrad's head and neck in alignment results in:

a. increased intrathoracic pressure


b. increased venous outflow
c. decreased venous outflow
d. increased intra abdominal pressure

58. Which of the following activities may increase intracranial


pressure (ICP)?

a. Raising the head of the bed


b. Manual hyperventilation
c. Use of osmotic Diuretics
d. Valsava's maneuver

59. After you assessed Conrad, you suspected increased ICP! Your
most appropriate respiratory goal is to:

a. maintain partial pressure of arterial 02 (PaO2) above 80 mmHg


b. lower arterial pH
c. prevent respiratory alkalosis
d. promote CO2 elimination
60. Conrad underwent craniotomy. As his nurse; you know that
drainage on a craniotomy dressing must be measured and marked.
Which findings should you report immediately to the surgeon?

a. Foul-smelling drainage
b. yellowish drainage
c. Greenish drainage
d. Bloody drainage

Situation 13 -As a Nurse, you have specific responsibilities as


professional. You have to demonstrate specific competencies.

61. The essential components of professional nursing practice are


all the following EXCEPT:

a. Culture
b. Care
c. Cure
d. Coordination

62. You are assigned to care for four (4) patients. Which of the
following patients should you give first priority?

a. Grace, who is terminally ill with breast cancer


b. Emy, who was previously lucid but is now unarousable
c. Aris, who is newly admitted and is scheduled for an executive check-
up
d. Claire, who has cholelithiasis and is for operation on call

63. Brenda, the Nursing Supervisor of the intensive care unit


(ICU) is not on duty when a staff nurse committed a serious
medication error. Which statement accurately reflects the
accountability of the nursing supervisor?

a. Brenda should be informed when she goes back on duty


b. Although Brenda is not on duty, the nursing supervisor on duty
decides to call her if time permits
c. The nursing supervisor on duty will notify Brenda at home
d. Brenda is not duty therefore it is not necessary to inform her

64. Which barrier should you avoid, to manage your time wisely?

a. Practical planning
b. Procrastination
c. Setting limits
d. Realistic personal expectation
65. You are caring for Vincent who has just been transferred to
the private room. He is anxious because he fears he won't be
monitored as closely as he was in the Coronary Care Unit. How can
you allay his fear?

a. Move his bed to a room far from nurse's station to reduce


b. Assign the same nurse to him when possible
c. Allow Vincent uninterrupted period of time
d. Limit Vincent's visitors to coincide with CCU policies

Situation 14 - As a nurse in the Oncology Unit, you have to be prepared


to provide efficient and effective care to your patients.

66. Which one of the following nursing interventions would be most


helpful in preparing the patient for radiation therapy?

a. Offer tranquilizers and antiemetics


b. Instruct the patient of the possibility of radiation burn
c. Emphasis on the therapeutic value of the treatment
d. Map out the precise course of treatment

67. What side effects are most apt to occur to patient during
radiation therapy to the pelvis?

a. Urinary retention
b. Abnormal vaginal or perineal discharge
c. Paresthesia of the lower extremities
d. Nausea and vomiting and diarrhea

68. Which of the following can be used on the irradiated skin


during a course of radiation therapy?

a. Adhesive tape
b. Mineral oil
c. Talcum powder
d. Zinc oxide ointment

69. Earliest sign of skin reaction to radiation therapy is:

a. desquamation
b. erythema
c. atrophy
d. pigmentation

70. What is the purpose of wearing a film badge while caring for
the patient who is radioactive?
a. Identify the nurse who is assigned to care for such a patient
b. Prevent radiation-induced sterility
c. Protect the nurse from radiation effects
d. Measure the amount of exposure to radiation

Situation 15 - In a disaster there must be a chain of command in place


that defines the roles of each member of the response team. Within the
health care group there are pre-assigned roles based on education,
experience and training on disaster.

71. As a nurse to which of the following groups are you best


prepared to join?

a. Treatment group
b. Triage group
c. Morgue management
d. Transport group

72. There are important principles that should guide the triage
team in disaster management that you have to know if you were to
volunteer as part of the triage team. The following principles
should be observed in disaster triage, EXCEPT:

a. any disaster plan should have resource available to triage at each


facility and at the disaster site if possible
b. make the most efficient use of available resources
c. training on disaster is not important to the response in the event of
a real disaster because each disaster is unique in itself
d. do the greatest good for the greatest number of casualties

73. Which of the following categories of conditions should be


considered first priority in a disaster?

a. Intracranial pressure and mental status


b. Lower gastrointestinal problems
c. Respiratory infection
d. Trauma

74. A guideline that is utilized in determining priorities is to


assess the status of the following, EXCEPT?

a. perfusion
b. locomotion
c. respiration
d. mentation
75. The most important component of neurologic assessment is:

a. pupil reactivity
b. vital sign assessment
c. cranial nerve assessment
d. level of consciousness/responsiveness

Situation 16 - You are going to participate in a Cancer Consciousness


Week. You are assigned to take charge of the women to make them aware of
cervical cancer. You reviewed its manifestations and management.

76. The following are risk factors for cervical Cancer EXCEPT:

a. immunisuppressive therapy
b. sex at an early age, multiple partners, exposure to socially
transmitted diseases, male partner's sexual habits
c. viral agents like the Human Papilloma Virus
d. smoking

77. Late signs and symptoms of cervical cancer include the


following EXCEPT:

a. urinary/bowel changes
b. pain in pelvis, leg of flank
c. uterine bleeding
d. lymph edema of lower extremities

78. When a panhysterectomy is performed due to cancer of the


cervix, which of the following organs are moved?

a. the uterus, cervix, and one ovary


b. the uterus, cervix, and two-thirds of the vagina
c. the uterus, cervix, tubes and ovaries
d. the uterus and cervix

79. The primary modalities of treatment for Stage 1 and IIA


cervical cancer include the following:

a. surgery, radiation therapy and hormone therapy


b. surgery
c. radiation therapy
d. surgery and radiation therapy

80. A common complication of hysterectomy is:

a. thrombophlebitis of the pelvic and thigh vessels


b. diarrhea due to over stimulating
c. atelectasis
d. wound dehiscence

Situation 17 - The body has regulatory mechanism to maintain the needed


electrolytes. However there are conditions/surgical interventions that
could compromise life. You have to understand how management of these
conditions are done.

81. You are caring for Leda who is scheduled to undergo total
thyroidectomy because of a diagnosis of thyroid cancer. Prior to
total thyroidectomy, you should instruct Leda to:

a. Perform range and motion exercises on the head and neck


b. Apply gentle pressure against the incision when swallowing
c. Cough and deep breath every 2 hours
d. Support head with the hands when changing position

82. As Leda's nurse, you plan to set up an emergency equipment at


her beside following thyroidectomy. You should include:

a An airway and rebreathing tube


b. A tracheostomy set and oxygen
c. A crush cart .with bed board
d. Two ampules of sodium bicarbonate

83. Which of the following nursing interventions is appropriate


after a total thyroidectomy?

a. Place pillows under your patient's shoulders


b. Raise the knee-gatch to 30 degrees
c. Keep your patient in a high-fowler's position
d. Support the patient's head and neck with pillows and sandbags

84. If there is an accidental injury to the parathyroid gland


during a thyroidectomy which of the following might Leda develops
postoperative?

a. Cardiac arrest
b. Dyspnea
c. Respiratory failure
d. Tetany

85. After surgery Leda develops peripheral numbness, tingling and


muscle twitching and spasm. What would you anticipate to
administer?

a. Magnesium sulfate
b. Calcium gluconate
c. Potassium iodine
d. Potassium chloride

Situation 18 - NURSES are involved in maintaining a safe and health


environment. This is part of quality care management.

86. The first step in decontamination is:

a. to immediately apply a chemical decontamination foam to the area of


contamination
b. a thorough soap and water was and rinse of the patient
c. to immediately apply personal protective equipment
d. removal of the patients clothing and jewelry and then rinsing the
patient with water

87. For a patient experiencing pruritus, you recommend which type


of bath:

a. Water
b. colloidal (oatmeal)
c. saline
d. sodium bicarbonate

88. Induction of vomiting is indicated for the accidental


poisoning patient who has ingested.

a. rust remover
b. gasoline
c. toilet bowl cleaner
d. aspirin

89. Which of the following term most precisely refer to an


infection acquired in the hospital that was not present or
incubating at the time of hospital admission?

a. Secondary bloodstream infection


b. Nosocomial infection
c. Emerging infectious disease
d. Primary bloodstream infection

90. Which of the following guidelines is not appropriate to


helping family members cope with sudden death?

a. Obtain orders for sedation of family members


b. Provide details of the factors attendant to the sudden death
c. Show acceptance of the body by touching it and giving the family
permission to touch
d. Inform the family that the patient has passed on

Situation 19 - As a nurse you are expected to participate in initiating


or participating in the conduct of research studies to improve nursing
practice. You have to be updated on the latest trends and issues
affecting profession and the best practices arrived at by the profession

91. You are interested to study the effects of meditation and


relaxation on the pain experienced by cancer patients. What type
of variable is pain?

a. Dependant
b. Correlational
c. Independent
d. Demographic

92. You would like to compare the support system of patients with
chronic illness to those with acute illness. How will you best
state your problem?

a. A descriptive study to compare the support system of patients with


chronic illness and those with acute illness in terms of demographic
data and knowledge about interventions
b. The effect of the Type of Support system of patients with chronic
illness and those with acute illness
c. A comparative analysis of the support: system of patients with
chronic illness and those with acute illness
d. A study to compare the support system of patients with chronic
illness and those with acute illness

93. You would like to compare the support, system of patients with
chronic illness to those with acute illness. What type of research
it this?

a. Correlational
b. Descriptive
c. Experimental
d. Quasi-experimental

94. You are shown a Likert Scale that will be used in evaluating
your performance in the clinical area. Which of the following
questions will you not use in critiquing the Likert Scale?

a. Are the techniques to complete and score the scale provided?


b. Are the reliability and validity information on the scale described?
c. If the Likert Scale is to be used for a study, was the development
process described?
d. Is the instrument clearly described?

95. In any research study where individual persons are involves,


it is important that an informed consent for the Study is
obtained. The following are essential information about the
consent that you should disclose to the prospective subjects
EXCEPT:

a. Consent to incomplete disclosure


b. Description of benefits, risks and discomforts
c. Explanation of procedure
d. Assurance of anonymity and confidentiality,

Situation 20 - Because severe burn can affect the person's totality it


is important that you apply interventions focusing on the various
dimensions of man. You also have to understand the rationale of the
treatment.

96. What type of debribement involves proteolytic enzymes?

a. Interventional
b. Mechanical
c. Surgical
d Chemical

97. Which topical antimicrobial is most frequently used in burn


wound care?

a. Neosporin
b. Silver nitrate
c. Silver sulfadiazine
d. Sulfamylon

98. Hypertrophic burns scars are caused by:

a. exaggerated contraction
b. random layering of collagen
c. wound ischemia
d. delayed epithelialization

99. The major disadvantage of whirlpool cleansing of burn wounds


is:

a. patient hypothermia
b. cross contamination of wound
c. patient discomfort
d. excessive manpower requirement

100. Oral analgecis are most frequently used to control burn


injury pain:

a. upon patient request


b. during the emergent phase
c. after hospital discharge
d. during the cute phase

ANSWER KEY:

1. A
2. D
3. D
4. B
5. D
6. A
7. B
8. A
9. D
10. A
11. B
12. B
13. B
14. C
15. B
16. C
17. C
18. C
19. A
20. C
21. C
22. A
23. A
24. D
25. A
26. D
27. A
28 D
29. C
30. D
31. D
32. D
33. B
34. C
35. D
36. B
37. D
38. B
39. C
40. C
41. B
42. C
43. D
44. D
45. A
46. D
47. A
48. C
49. B
50. A
51. C
52. A
53. D
54. A
55. D
56. A
57. B
58. B
59. D
60. A
61. A
62. B
63. A
64. B
65. B
66. C
67. A
68. D
69. B
70. C
71. B
72. C
73. D
74. B
75. D
76. A
77. B
78. C
79. D
80. A
81. C
82. B
83. C
84. D
85. B
86. C
87. B
88. D
89. B
90. A
91. A
92. C
93. A
94. A
95. A
96. D
97. B
98. A
99. A
100. C

Anda mungkin juga menyukai